Вы находитесь на странице: 1из 125

1 ENGINEERING MECHANICS [SUBJECT CODE-ME101] FIRST YEAR FIRST SEMESTER STUDY MATERIAL 2 SYLLABUS MODULE 1 Importance of Mechanics in engineering;

Introduction to Statics; Concept of Particle and Rigid Body; Types of forces: collinear, concurrent, parallel, concentrated, distributed; Vector and scalar quantities; Force is a vector; Transmissibility of a force (sliding vector). Introduction to Vector Algebra; Parallelogram law; Addition and subtraction of vectors; Lami s theorem; free vector; Bound vector; Representation of forces in terms of i, j, k; Cross product and Dot product and their applications. Two dimensional force system; Resolution of forces; Moment; Varignon s theorem; Couple; Resolution of a coplanar force by its equivalent force-couple system; Resultant of forces. MODULE 2 Concept and Equilibrium of forces in two dimensions; Free body concept and diagram; Equations of equilibrium. Concept of Friction; Laws of Coulomb friction; Angle of Repose; Coefficient of friction MODULE 3 Distributed Force: Centroid and Centre of Gravity; Centroid of a triangle, circular sector, quadrilateral, composite areas consisting of above figures. Moments of inertia: MI of plane figure with respect to an axis in its plane, MI of plane figure with respect to an axis perpendicular to the plane of the figure; Parallel axis theorem; Mass moment of inertia of symmetrical bodies e.g. cylinder, sphere, cone. Concept of simple stresses and strains: Normal stress, Shear stress, bearing stress, Normal strain, Shearing strain; Hooke s law; Poisson s ratio; Stress-strain diagram of ductile and

brittle materials; Elastic limit; Ultimate stress; Yielding; Modulus of elasticity; Factor of safety. MODULE 4 Introduction to Dynamics: Kinematics and Kinetics; Newton s laws of motion; Law of gravitation & acceleration due to gravity; Rectilinear motion of particles; determination of position, velocity and acceleration under uniform and non-uniformly accelerated rectilinear motion; construction of x-t, v-t and a-t graphs. Plane curvilinear motion of particles: Rectangular components (Projectile motion); Normal and tangential components (circular motion). Kinetics of particles: Newton s second law; Equation of motion; D.Alembert s principle and free body diagram; Principle of work and energy; Principle of conservation of energy; Power and efficiency. 3 LECTURE CONTENTS SL Lecture No. Lecture Title Page 1. Lecture 1 Introduction to mechanics, Statics, Concept of Particle and Rigid body, Force, Transmissibility of force 6-7 2. Lecture 2 Classification and type of forces 8-11 3. Lecture 3 Introduction to Vector Algebra, Parallelogram Law, Addition and Subtraction of vector 12-16 5. Lecture 4 Lami s theorem, Free vector, Bound vector, Representation of forces in term

of I,j & k 17-18 5. Lecture 5 Cross product, Dot product, examples 19-20 6. Lecture 6 Examples 21-22 7. Lecture 7 Two dimensional force system, Resolution of forces, Moment, Varignon s theorem 23-24 8. Lecture 8 Couple, Example 25-29 9. Lecture 9 Resolution of a coplanar force by its equivalent force-couple system, Resolution of forces, Example 30-33 10. Lecture10 Example 34-38 11. Lecture11 Concept and equilibrium of forces, Free body diagram 39-40 12. Lecture12 Free body diagram , example 41-42 13 Lecture13 Equation of equilibrium, example 43-49 14 Lecture14 Concept of friction, Columb law of friction, Angle of repose 50-54 4 15 Lecture15 Coefficient of friction, Kinetic friction 55-59 16 Lecture16 Examples 60-68 17 Lecture17 Distributed force, Centroid & Centre of Gravity

69-73 18 Lecture18 Determination of Centroid & C.G. 74-78 19 Lecture19 Centroid of an arc, Centroid of a triangle, Example 79-82 20 Lecture20 Examples 83-86 21 Lecture21 Moment of Inertia, Parallel axis theorem 87-90 22 Lecture22 Moment of inertia of plain figure, Perpendicular axis theorem, Radius of gyration 91-97 23 Lecture23 Examples 98-101 24 Lecture24 Concept of stress, different types of stresses & strains, Hook s law 102-110 25 Lecture25 Possoin ratio,Stress-Strain curve for ductile & brittle material, Example 111-117 26 Lecture26 Projectiles 118-120 27 Lecture27 Projectiles at inclined plane 121-126 28 Lecture28 Examples of Projectiles, Kinematics, Rectilinear motion 127-129 29 Lecture29 Relative motion and example 130-138 30 Lecture30 Kinetics of particle, curvilinear motion, example 139-141 5

31 Lecture31 Rolling motion 142-145 32 Lecture32 Solution to kinetic problem, 146-148 33 Lecture33 Solution to problems 149-151 34 Lecture34 Example 152-157 35 Lecture35 Work, Power, Energy 158-163 36 Lecture36 Impulse and Momentum 164-169 6

Lecture 1 : Introduction to Mechanics

STATICS Statics is the branch of mechanics concerned with the analysis of loads (force, torque/moment) on physical systems in static equilibrium, that is, in a state where the relative positions of subsystems do not vary over time, or where components and structures are at a constant velocity. When in static equilibrium, the system is either at rest, or its center of mass moves at constant velocity. The study of moving bodies is known as dynamics. By Newton's first law, this situation implies that the net force and net torque (also known as moment of force) on every body in the system is zero. From this constraint, such quantities as stress or pressure can be derived. The net forces equaling zero is known as the first condition for equilibrium, and the net torque equaling zero is known as the second condition for equilibrium. See statically determinate. Solids Statics is used in the analysis of structures, for instance in architectural and structural engineering. Strength of materials is a related field of mechanics that relies heavily on the application of static equilibrium. A key concept is the centre of gravity of a body at rest: it represents an imaginary point at which all the mass of a body resides. The position of the point relative to the foundations on which a body lies determines its stability towards small movements. If the center of gravity falls outside the foundations, then the body is unstable

because there is a torque acting: any small disturbance will cause the body to fall or topple. If the centre of gravity falls within the foundations, the body is stable since no net torque acts on the body. If the center of gravity coincides with the foundations, then the body is said to be met stable. Particle: A body of infinitely small volume i.e. negligible dimensions but having mass concentrated at a point is called particle. Such a body cannot exist theoretically. Rigid Body: A rigid body may be defined as a body in which the relative positions of any two particles do not change under the action of forces. Nobody is perfectly rigid. A body when subjected to external forces, it must undergo some form of deformation, however small it may be. 7 Force It is the agent which changes or tends to change the state of rest or motion of a body. The force is a vector quantity. The characteristics of force are magnitude, direction and point of application. The unit of force is N , KN Kgf etc. Principle of transmissibility of Forces The state of rest or of a motion of a rigid body is unaltered; if a force acting on a body is replaced by another force of the same magnitude and direction but acting anywhere on the body along the line of action of the replaced force. If deformation of the body is to be considered, the law of transmissibility will not hold good. By transmission of force, only the state of the body is unaltered, but not the internal stresses which may develop in the body. This force is a sliding vector as this has a unique line of action in space but not a unique point of application. 8 Lecture 2 : Classification & Types Of Forces

1.1.1 CLASSIFICATION OF FORCE The general classification of force is presented below:

Gravitational force

Electromagnetic force

Strong force

Weak force

But engineering mechanics point of view, a system of forces can be classified as: System of Forces Coplanar Forces (lying on the same plane) Coplanar forces may be Collinear Concurrent Non-Concurrent Parallel Force Concurrent Non-Concurrent Parallel Non-Parallel 9 Non-Parallel Table 1.2 TYPES & CHARACTERISTICS OF FORCE SYSTEM Force System Line Diagram Characteristics Coplanar forces Non-Coplanar Forces (do not lie on the same) plane) Non-Coplanar forces may be

Y x Lines of action of all forces lie on the same plane Non-Coplanar forces F1 y

x F z fF F2

Lines of action of all forces do not lie on the same plane Collinear Forces Lines of action of all forces lie on the same st. line Concurrent Forces Lines of action of all forces pass thru the same point 10 Parallel Forces F1 F2 F3 Lines of action all forces are parallel to each other According to the effect produced by the force a) External Force This force is applied to externally to a body. b) Internal Force This force is developed into the body to resist the deformation or change of shape of a body. c) Active Force This force causes a body to move or to change its shape. d) Passive Force This force represents the motion of a body. According to the nature of force

a) Action or Reaction Force When two bodies come into contact with each other, each body will exert a force on the other body. Out of these forces one is known as action and other is known as reaction. It is seen that action and reaction are equal. b) Attraction and Repulsion The non-contact force exerted by one body on the another body without any visible medium are known as attraction or repulsion force e.g. magnetic force. c) Tension or Compression When a pull is applied on a structural member, it tends to elongate/increase in length, pull is known as tensile force. If a structure member is subjected to two equal and opposite pushes and the member tends to shorten/decrease in length, member is said to be in compression. According to the force applied to a point or over an area a) Concentrated Force: The point of application of this force is considered to be a point. 11 b) Distributed Force: The force which is distributed over an area, that force is known as distributed force. Multiple Choice Questions 1. Forces are called concurrent when their lines of action meet in a) One plane, b) One point, c) Different planes , d) Two points 2. Forces are called coplanar when all the forces acting on a body a) One plane, b) One point, c) Different planes, d) Different points 3. A force is completely defined when we specify a) Its magnitude, b) Its direction, c) Its point of application, d) All of the above 4. The amount of matter contained in a body is called its a) Mass, b) Volume, c) Weight, d) None of the above 5. Two non collinear parallel equal forces acting in opposite direction a) Balances each other, b) Constitute a couple, c) Constitute a moment d) Constitute a moment of couple 12

Lecture 3 : Introduction To Vector Algebra Introduction to Vector Algebra We use two kinds of quantities in mechanics-scalars and vectors. Scalar quantities are those with which only a magnitude is associated. Examples of scalar quantities are time, volume, density, speed energy and mass. Vector quantities, on the other hand, possess direction as well as magnitude, and must obey the parallelogram law of addition. Examples of vector quantities are displacement, velocity, acceleration, force, moment and momentum. Velocity is specified by a direction as well as speed. However, the mass of a body is completely specified by a magnitude, and hence mass is a scalar quantity. Since the weight of a body is the force with which it is attracted by the earth, weight has a downward direction and thus is a vector quantity. Since weight and mass are different physical concepts, they should not be measured in the same units. The gram is a unit of mass. The force with which the earth attracts a one-gram mass at a standard location sometimes is called a gram-weight of force. * Since weight is proportional to mass in any given locality, this experiment is not affected by the slight variations consequent to laboratory conditions Parallelogram Law of forces If two forces acting at a point be represented in magnitude, direction and sense by the two adjacent sides of a parallelogram, their resultant is represented in magnitude, direction and sense by the diagonal of the parallelogram passing through that point . In the figure vectors P and Q are represent in Magnitude, direction and sense by OA and OB respectively. The resultant R is represented by OC in magnitude, and direction. Q R B C

We now find magnitude of resultant R. O

P A D

From C we draw CD perpendicular to OA produced. Let = angle between two forces P and Q = <AOB (corresponding angle)

Now <DAC = <AOB =

In parallelogram OACB, AC is parallel and equal to OB So, AC = Q 13 In 2 = OD 2 +DC 2 Or, R 2 = (OA+AD) 2 +DC 2 Or, R 2 = OA 2 +2. OA.AD + AD 2 +DC 2 Or, R 2 OCD, OC

=P 2 +2PQcos +Q 2 cos 2 +Q 2 sin 2

Or, R = (P 2 +Q 2 +2PQcos ) Direction of resultant vector R Let = angle made by resultant with OA Then from So, = tan 1 [Q sin / P+Qcos ] Composition and Resolution of Concurrent Forces by Vector Methods In order to add scalar quantities, one has merely to make the algebraic addition. When one wishes to add two vector quantities, the process is more difficult because their directions must be considered. The vector sum of two vector quantities is the single vector quantity that would produce the same result as the original pair. OCD, tan = CD/OD = Q sin / P+Qcos

The addition of vector quantities is greatly simplified by representing the vector quantity graphically. A vector is the line segment whose length represents the magnitude of a vector quantity and whose direction is that of the vector quantity. The sense along the line is indicated by an arrow. For example, a force of 100lb. acting at an angle of 30 above the horizontal may be represented by the line OA. Fig. 1, which is 5 units long and has the correct direction. Each unit of length thus represents 20lb. When vectors do not have the same line of action, their vector sum is not their algebraic sum but a geometric sum. 14 This geometric sum may be determined by either graphical or analytical methods. Graphical methods are simple and direct but are limited in precision to that obtainable by drawing instruments. Analytical methods have no such inherent limitations. In this experiment both graphical and analytical methods will he applied to forces as examples of vector quantities, but the same methods apply to all vector quantities. The vector sum, or resultant, of a set of forces is the single force that will have the same effect, Vector Summation by Graphical Methods: As an example of vector addition let us consider the case of two forces acting on a body in such a direction that the forces are concurrent, that is their lines of action, if projected would intersect at a point. The vectors OA and OB representing two such forces are shown in Fig. 2. Their vector sum or resultant R, is found by constructing a parallelogram having the two vectors as sides and drawing the concurrent diagonal, as shown in Fig. 3. This diagonal vector R represents in magnitude and direction the single force that is equivalent to the origina1 pair that is their vector sum. When the resultant of more than two vectors is to be obtained graphically a polygon method is used. This is illustrated in Fig. 4. The vector A is first constructed by the use of a chosen scale and reference direction. Then, from the head of A, the vector B is drawn. It is clear that the vector M is the resultant of vectors A and B, since M would be the concurrent diagonal of a

parallelogram if such a parallelogram had been drawn, as was done in Fig. 3. Similarly, it follows that the vector R is the resultant of M and C or of A, B, and C. When the resultant of several forces is required this method is simpler than the parallelogram method. It should be noted that when the parallelogram method is used, the arrows, with their tails together, all radiate from a common point. But in the polygon method the tail of the second arrow coincides with the head of the first, etc. 15 Summation of Vectors by Analytical Methods: The resultant of two vectors may be determined analytically by the use of the trigonometric laws of sines and cosines. Consider the vectors A and B in Fig. 5. The magnitude of the resultant R can be obtained by the application of the law of cosines: R 2 =A 2 + B2 + 2ABcos (1) The direction of R can then be obtained from the law of sines: Sin /Sin = B/R Since sin = sin , Sin = (B/R) sin (2) Components of Vectors: Any single force may be replaced by two or more forces whose joint action will produce the same effect as the single force. These various forces are said to be components of the single force. The most useful set of components is usually a pair at right angles to each other, as shown in

Fig. 6. The force B is the resultant of Forces Bx and By. Therefore conditions are unchanged by replacing the single force B by forces Bx and By, called their X- and Y- components. It is obvious from Fig. 6 that Bx = B cos and By = B sin . Component Method for Addition of Vectors: Fig. 7 illustrates the component method of computing the resultant of A, 16 B, and C. The X- axis is so chosen that it coincides with the vector A, and the vectors B and C are resolved into X- and Y-components. The three forces A, B, and C have been replaced by five forces (A has no Y- component). The slim of the component along either axis may be computed by algebraic addition. Calling the sum of the X-components Fx and the sum of the Ycomponents Fy, it follows that the resultant R is given by the equation R 2 = (FX) 2 + (FY) 2 (3) and that the angle - the angle that R makes with the Xaxis may be determined from the equation tan = Fy/Fx (4) 17 Lecture 4 : Lami s Theorem Lami s Theorem If a body is in equilibrium under the action of Q

three forces, each force is proportional to the sine of the angle between other two forces.

Suppose the three forces P, Q & R are acting at a point O and they are in equilibrium as shown in figure. According to the Lami s theorem, P/sin = Q/sin = R/sin R

Proof of Lami s theorem The three forces acting on a point are in Equilibrium and hence they can be represented by the three sides of a triangle taken in the same order. In figure .applying sine rule, we get P/ sin ( - ) = Q/sin ( - ) = R/sin ( - ) Or P/sin = Q/sin = R/sin P R ( - ) Q

Free Vector Situation in which vector may be positioned anywhere in space without loss or change of meaning provided that magnitude and direction are kept intact. It is not constrained to any particular direction and location. It can be moved anywhere in space without rotation. Example 1. The velocity of water particle (having turbulent motion). 18 2. Couple (the change in location of application couple in a plane does not change its effect) Bound Vector A bound or fixed vector has a definite point of application. It is specified by magnitude,

direction and its point of application. Change in the point of application of force will alter its effect. y Representation of forces in terms of I,j & k FY

Many problems in mechanics require analysis in three dimensions and for such problem it is often necessary to resolve a force into its three mutually perpendicular components. The force F acting at point O has the rectangular components FX, FY, FZ. FZ O FX x

z Fx = F cos , Fy = F cos , Fz = F cos Also F = (Fx 2 + FY 2 + FZ 2)

The cosines of ,

and are known as the direction cosine of vector F and denoted by

l= cos , m= cos , n= cos

The three angles are related by, cos 2

+cos 2 +cos 2 = 1 or l 2 +m 2 +n 2 =1 Let i= vector of unit length in the +tive x direction j= vector of unit length in the +tive y direction k= vector of unit length in the +tive z direction The force F is represented by, F = Fxi + FY j+ Fzk Magnitude of unit vector F = l F l = (Fx 2 + FY 2 + FZ 2 ) But Fx = F cos , Fy = F cos , Fz = F cos Substituting these values in earlier equation, we get

F = F cos i + F cos j+ F cos k 19

Lecture 5 : Cross Product & Dot Product Cross Product or Vector Product The cross product of vectors A & B is a vector quantity and is defined as the product of the magnitude of the vector A, the magnitude of the vector B and the sine of the smaller angle between the two vectors. Its direction is perpendicular to the plane containing the vector.

Now if n is the unit vector which gives the

direction of the resultant vector R,

R = A X B = lA l IB l sin . n = AB sin . n Dot Product or Scalar Product

Dot product of A and B is a scalar quantity and is defined as the product of the magnitude the vectors and cosines of their included angle.

A. B = lA l IB l cos = AB cos Following points is to be noted (i) When 0 , and vectors A & B are along same direction, A. B = AB cos0 = AB (ii) When 0 , and vectors A & B are perpendicular to each other, A. B = AB cos90 = 0 (iii) A. B = A times projection of B on A projection of A On B = 90 =0

= B times projection of A on B (iv) The angle between the vector A and B is O

Cos

=A. B / lAl lBl

Projection of B on A

= A. B/AB 20 The dot product is used to find the component of a vector along an arbitrary direction and to define the term work. Example.1 Add a 20N force in the positive x direction to a 50N force at an angle 45 to axis in the first quadrant and directed away from origin. F1 = 20N, F2 = 50N B We add vectorally, F1 + F2 = F To get the sum, we may use the law of cosines for the triangular portions of the parallelogram by using the A OCA, 50N 45 F C

O lFl = (20 2 + 50 2 + 2.20.50.cos45) = 65.68267N (Ans)

20N

The direction of the vector may be described by giving angle and sense. The angle is determined by employing the law of sine.

For Or = 32.566 (Ans)

OBA, 50/sin = 65, 68267/sin135

Example.2 Force A (given as a horizontal 10N force) and B (vertical) and add up to a force C that has magnitude of 20N. What is the magnitude of force B? y Let OA represents force A (FA) = 10N Let OB Let OC B (FB) =? C (FC) = 20N B OC 2 = OA 2 + OB 2 B Or OB= (20 210 2 ) = 17.32N Tan = 17.32/10 or = 60 21 Lecture 6 : Solved Examples Example3. FB 10N O A C B A C 20N

10N x 20N

A force vector of magnitude 100N has a line of action with direction cosines, l= 0.7, m=0.2 & n=0.59 relative to reference xyz. The vector points away from the origin. What is the component of the force vector along a direction a having direction cosines, l=-0.3, m=0.1 & n=0.95 for the xyz reference? F = 100 (li +mj+nk) = 100 (0.7i +0.2j+0.59k) = 70i+20j+59k Unit vector along a = li+mj+nk = -0.3i+0.1j+0.55k Component of F along a is Fcos = [F] [a] cos = F. a = (70i+20J+59k) (-0.3i+0.1j+0.55k) N = 37.05N Example.4 Making use of the cross product, give the unit vector n normal to the inclined surface ABC. Given OB=8cm, OC=10cm, <ABY=150 From figure we can write, <OBA=180-150=30, <BOA= 90 So AO = OB tan30 = 8 x tan30m =4.618m Coordinate of the points A,B &C are A(4.618,0,0), (0,8,0) , (0,0,10) respectively. z Fcos R

AB = -4.618i+8j+0k and AC = 4.618i+0j+10k

Area = .AB.AC = (-4.618i+8j+0k) (4.618i+0j+10k)

O B

=(40i+23.09j+18k x Unit Vector = (40i+23.09j+18k)/ (40 2 + 23.09 2 +18 2

A 150

) = 0.804i+.464j+.371k) (ans)

Assignment 1. Subtract the 20N force in example.1 from50N force. 400N 40 500N 22 2. What is the sum of forces transmitted by the structural rods to the pin at A? 3. Given the vectors A= 6i+3j+10k, B=2i-5j+5k, C=5i-2i+7k What vector D gives the following results D.A=20, D.B=5, D.C=10 Multiple Choice Questions 1. Moment of a force is a a) Scalar quantity, b) Vector quantity, c) Either a or b, d) None of the above 2. A vector having a unit magnitude is known as a) Null vector, b)Free vector, c)Unit vector,d)None of the above 3. The dot product of two orthogonal vectors is a) 1, b) o, c) No definite value, c) None of the above 4. Two vectors are said to be equivalent if they produces a) Same magnitudes, b) Same directions, c) Same effect in a certain respect d) None of these .

23 Lecture 7 : Two Dimensional Forces Two dimensional force system & Resolution of forces When several forces of different magnitudes and directions act upon a body, they constitute a

system of forces. If all the forces in a system lie in a single plane is known as coplanar force system. If all the forces in a system lie on the same plane and the lines of action of all forces do not pass through a single point, the system is known as coplanar non-concurrent force system. The most common two-dimensional resolution of a force vector is into rectangular components. It follows from the parallelogram rule that the vector F (in figure) may be written as F=FX+Fy FX Where Fx & Fy are vector components of F in the x and y directions. Each of the two vector components may be written as a scalar times the appropriate unit vector. In terms of the unit vector I and j, FX=Fxi & FY =Fyj and thus we may write F = Fxi +Fyj Where the scalars FX and FY are the x and y scalar components of the vector F. The scalar components can be positive or negative, depending on the quadrant into which F points. For the force vector shown above the x and y scalar components are both positive are related to the magnitude and direction of F by FX = Fcos & FY = Fsin 2 + Fy 2 ), 1 (FX/ FY) Moment of a force = tan F = (FX I FY x F y j

The product of the magnitude of a force and the perpendicular distance of the line of action the force from appoint is known as moment of a force about that point. moment of a force about a point is the measure of its rotational effect. The rotational effect of a force becomes very important when we deal with non concurrent force system. 24 Moment M of a force P about point O as shown in figure is given by, M = P x d where d is the perpendicular distance between the line of action of the force and the moment centre. The tendency of this moment is to rotate the body in anticlockwise direction about O. Varignon's theorem One of the most useful principles of mechanics is Varignon s theorem. The algebraic sum of the moments of a system of coplanar forces about a moment centre in their plane is equal to the moment of their resultant force about the same moment centre. Proof According to the figure, let F be the Resultant of the forces F1 and F2 acting at A. Let us consider any point B lying in the plane of the forces, as a moment centre. Let I, l1 and l2 be the moment arms of the forces F, F1 and F2 respectively from the moment centre B. F2 F Y P d .O

2 We join AB and consider it as y-axis And draw x-axis at right angle to it at A as shown in figure. O FX2 FX1 FX F1 1 X

Now moment of the force F about B = F X l = F (BAcos ) = BA (Fcos ) = BAFX Moment of the force F1 about B = F1 X l1 = F1 (BAcos 1) = BA (F1 cos 1) = BAFX1 .1 Moment of the force F2 about B = F2 X l2 = F1 (BAcos 2) = BA (F2 cos 2) = BAFX2.....2 Now adding equation 1 and 2, F1 X l1 + F2 X l2 = BA (FX1 + FX2) But the sum of the x components of forces F1 and F2 = x components of the resultant F So FX = FX1 + FX2 or BAFx = BA (FX1 + FX2) So from Fx l = F1 X l1 + F2 X l2 hence proved If a system of forces consists of more than two forces, the above proof can be extended by taking into consideration all forces. 25 Lecture 8 : Couples Couple Two parallel forces of equal magnitude but opposite in direction are separated by a finite distance to form a couple. The algebraic sum of the forces forming a couple is zero, which means the translatory effect is zero. The algebraic sum of the moments of the two forces of a couple is independent of the position in the d2 1 . d1 P d P

plane of the couple, of the moment centre and is equal to the moment centre and is always equal to the product of the magnitude of the forces and the perpendicular distance between the two forces. Let the magnitude of the forces forming a couple is P and the perpendicular distance between the two forces is d. Considering the moment of the two forces forming a couple about point 1as shown. Le t the moment be M1. So M1 = Pd2 Pd1 =P (d2 d1) = P d

d3

d4

Now we consider the moment of the forces about point 2 as shown. Let M2 is the moment, then M2 = P (d3+d4) = P. d The moment of a couple about any point is the same. Since the only effect of a couple is a moment (rotational effect) and this moment is the same about any point, the action of a couple is unchanged if (a) The perpendicular distance of the forces i.e. arm is turned in the plane of couple through any angle about one of its end. (b) The magnitude of the forces and the arm of the couple both are changed in such a way that the moment of the couple remains unchanged. (c) The couple is shifted to any other position. 26 Example 1: A person is holding a 100N weight (that is roughly a 10kg mass) by a light weight (negligible mass) rod AB. The rod is 1.5m long and weight is hanging at a distance of 1m from the end A, which is on a table (see figure 6). How much force should the person apply to hold the weight? Let the normal reaction of the table on the rod be N and the force by the point be F1. Then the two equilibrium conditions give

27 (Note: If the brick did not provide friction, the force applied cannot be only in the vertical direction as that would not be sufficient to cancel the horizontal component of N). Let us see what happens if the brick offered no friction and we applied a force in the vertical direction. The fulcrum applies a force N perpendicular to the rod so if we apply only a vertical force, the rod will tend to slip to the left because of the component of N in that direction. Try it out on a smooth corner and see that it does happen. However, if the friction is there then the rod will not slip. Let us apply the equilibrium conditions in such a situation. The balance of forces gives Let us choose the fulcrum as the point about which we balance the torque. It gives Then The normal force and the frictional force can now be calculated with the other two equations obtained above by the force balance equation. Example2: To balance a heavy weight of 5000 N, two persons dig a hole in the ground and put a pole of length l in it so that the hole acts as a socket. The pole makes an angle of 30 from the ground. The weight is tied at the mid point of the pole and the pole is pulled by two horizontal ropes tied at its ends as shown in figure 2. Find the tension in the two ropes and the reaction forces of the ground on the pole. 28 To solve this problem, let me first choose a co-ordinate system. I choose it so that the pole is over the y-axis in the (y-z) plane (see figure 2). The ropes are in (x.y) direction with tension T in each one of them so that tension in each is written as You may be wondering why I have taken the tension to be the same in the two ropes. Actually it arises from the torque balance equation; if the tensions were not equal; their component in the xdirection will give a nonzero torque. Let the normal reaction of the ground be (Nx, Ny, Nz). Then the force balance equation gives Taking torque about point O and equating it to zero, we get which gives 29

Lecture 9 : Resolution Of Coplanar Forces

Resolution of a coplanar force by its equivalent force-couple system The effect of force acting on a body is the tendency to push or pull the body in the direction of the force, and to rotate the body about any fixed axis which does not intersect the line of force. We can represent this dual effect more easily by replacing the given force by an equal parallel force and a couple to compensate for the change in the moment of the force. This replacement is illustrated in figure where the given force F acting at point A is replaced by an equal force F and at some point B and the counterclockwise couple M= F d. B .B A. F F A d F F

Resultant of forces The most common type of force system occurs when the forces all act in a single plane, say, the x-y plane. We obtain the magnitude and direction of the resultant force R by forming the force polygon. Thus for any system of coplanar forces we may write R = F1 +F2 + F3 + .= F

Rx = FX RY = FY where R = [( FX) + ( FY) ]

= tan 1 R Y/ Rx = tan 1 Example 3:

FY / FX 30

The picture below shows the forces acting on a parked car. If the weight of the car acts exactly halfway between the two wheels and the weight is 1000lbs how much force is exerted on the rear wheel? What about the front wheel?

Writing the force equations

There are no forces in the x direction Writing moment equation about front wheel Subbing RB back into the sum Fy Please note that Rf and RB are distributed over two wheels. Each front wheel supports half of Rf and each back wheel supports half of RB. Example 4. A weight of 15kN hangs from a point C, by two strings BC and AC (Fig. to Exmpl.5.1). Determine the tensions in the strings. B B C B T1 T2

75 135 150 A 15KN 31 The load 15kN force acts downwards inducing tensions T1 and T2 in the strings CB & CA (Fig to Exmpl.5) Applying Lami s theorem: = Or, T1 = 15kN (sin150 o ) / sin75 o = 7.76kN Ans. sin =

& o

T2 = 15kN (sin 135

) / sin 75 o = 10.98kN Ans.

Example 5. ABCD is a square of 1m side. It is being acted upon by a number of forces as shown (Fig. to Exampl.4.3). The system is at equilibrium. Find the magnitude of i) P & Q ii)The resultant couple. P DC Q 200kN AB 100KN 1m Fig. to Example. 5

Resolving the system of forces horizontally and computing H = 0 100kN 100kN (cos45 o ) P=0 Or, P = 29.3kN ns.

Resolving the forces vertically and computing V = 0 200kN 100kN (sin45 o ) Q=0 Or, 15kN Q = 129.3kN Ans.

sin75 o T1 sin150 o T2 sin135 o32 ii) Couple Moment of a couple = M of constituents forces about any point MA = + 200kN x 1m + P x 1m = 200 + 29.3 kN = 229.3kN Ans. Since the moment is + tive, the couple is counterclockwise. Example 6. At what point on the beam a weight of 2kN is to be placed so that one of the strings may just snap? The weight of the beam 4kN acts halfway between A and B. TA TB 3m x A D C B

AC = BC

2kN 4kN Fig to Example 6 33 TA and TB are the tensions in the two strings, which balance the net downward force.

When either TA or, TB becomes 3.5kN, the corresponding string snaps. Let string at end A snaps. Taking moment about B and for balance: MB = 0 3.5kN x 3m = 2kN x (3 x) + 4kN x 1.5m Or, x = 0.75m Ans.

34 Lecture 10 : Solved Examples Example7.The screw eye in Fig. is subjected to two forces. Determine the magnitude and direction of the resultant force.

10

Parallelogram Law. The parallelogram law of addition is shown in Figure Two unknowns are the magnitude of FR and the angle (theta). FR = (100 N) 2 + (150 N) 2 2x100 Nx150 N cos 115 =213N (Ans)

The angle is determined by applying the law of sines, using the computed value of FR. 150 N 212.6 N

--------- = sin

-----------sin 115

150 N sin = ------------- (0.9063) 212.6 N F2=150N F1= 100N 15 15 10

90-25=65 125 65 150N 100N FR 35 = 39.8

Thus, the direction (phi) of FR, measured from the horizontal, is = 39.8 + 15.0 = 54.8 (ans) Example8.The force F acting on the frame shown in Figure has a magnitude of 500 N and is to be resolved into two components acting along members AB and AC. Determine the angle measured below the horizontal, so that the component is directed from A toward C and has a magnitude of 400 N. By using the parallelogram law, the vector addition of the two components yielding the resultant is shown in Figure A. Note carefully how the resultant force is resolved into the two

components FAB and FAC which have specified lines of action. The angle can be determined by using the law of sines. The corresponding vector triangle is shown in Fig. 400 N 500N

---------- = ------sin sin 60

400 N sin = ---------- x sin 60 = 0.6928 or = 43.9 500 N Hence, = 180 - 60 - 43.9 = 76.1 (ans)

Using this value for

we apply the law of cosines or the law of sines to find the value of FAB

which has a magnitude of 561N .

A B C 500N

30 FAC=400N FAB 500N

60 30

FAC=400 500N FAB 60 FIG A FIG B36 Example9. The system in Figure shown below is in equilibrium with the string in the center exactly horizontal. Find (a) tension T1, (b) tension T2, (c) tension T3 and (d) angle .

T3 T1 T2 (b) 40N (a) 50N T2

(a) Forces at the left junction of the strings. (b) Forces acting at the right junction of the strings.. We have to solve for four unknowns (T1, T2, T3 and ). We consider the points where the strings meet; the left junction is shown in Figure (a). Since a string under tension pulls inward along its length with a force given by the string tension, the forces acting at this point are as shown. Since this junction in the strings is in static equilibrium, the (vector) sum of the forces acting on it must give zero. Thus the sum of the x components of the forces is zero: T1 sin 35_ + T2 = 0 1

The sum of the y components of the forces is zero: +T1 cos 35- 40N = 0 2

Now we look at the right junction of the strings; the forces acting here are shown in

Figure (b). Again, the sum of the x components of the forces is zero: T2 + T3 sin = 0 3

The sum of the y components of the forces is zero: +T3 cos 50N = 0 .4

And at this point we are done with the physics because we have four equations for four 40N 50N T1 T2 T3 35 37 unknowns. We will do algebra to solve for them. From Eq. 2, we get, T1 = 40N/cos 35 = 48.8N and then Eq. 1 gives us T2, 2 = T1 sin 35 = (48.8N) sin 35 = 28.0N . We now rewrite Eq. 3. As: T3 sin = T2 = 28.0N And Eq. 4 as: T3 cos = 50.0N .6 ..5

Now if we divide the left and right sides of 5 by the left and right sides of 6 we get: tan = (28.0N)/(50.0N) = 0.560 And then = tan 1(0.560) = 29.3

Finally, we get T3 from Eq. 3.9: T3 = 50.0Ncos 29.3 = 57.3N Summarizing, we have found: T1 = 48.8N, T2 = 28.0N, T3 = 57.3N, = 29.3 Assignment 1A. The vertical force F acts downward at A on the two-membered frame. Determine the magnitudes of the two components of F directed along the axes of AB and AC. Set F=500N, <ABC=45, <ACB=60 1B. Solve Prob. 1A with F = 350 lb.

2. For the stepladder shown in Figure, sides AC and CE are each 8.0 ft long and hinged at C. Bar BD is a tie rod 2.5 ft long, halfway up. A man weighing 192 lb climbs 6.0 ft along the ladder. Assuming that the floor is frictionless and neglecting the weight of the ladder, find (a) the tension the tie rod and the forces exerted on the ladder by the floor at (b) A and (c) E. Hint: It will help to isolate parts of the ladder in applying the equilibrium conditions. Multiple Choice Questions 1. A force can be replaced by a) A force of same magnitude and couple b) A force-couple combination so that equilibrium is maintained c) A force of different magnitude and couple d) None of the above A B C C D38 2. Two non-collinear parallel equal forces acting in opposite direction a) Balance each other b) Constitute a couple c) Constitute a moment d) None of the above 3. The force polygon for coplanar concurrent forces a) Must Close b) Must not close A E B in

c) Anyone of the above d) None of the above 4. Moment of a force a) Varies directly with its distance from the pivot b) Varies inversely with its distance from the pivot c) Is independent of its distance from the pivot d) None of the above 5. A couple consists of a) Two like parallel forces of same magnitude b) Two like parallel forces of different magnitude c) Two unlike parallel forces of same magnitude d) Two unlike parallel forces of different magnitude 39 Lecture 11 : Concept Of Equillibrium Equilibrium: Many problems that concern the physicist and engineer involve several forces acting on a body under circumstances in which they produce no change in the motion of the body. This condition is referred to as equilibrium. The body does not necessarily have to be at rest, but its motion must retain the same velocity; hence both magnitude and direction of motion are unchanged. First Condition for Equilibrium: Insofar as linear motion is concerned, a body is in equilibrium if there is no resultant force acting upon it that is if the vector sum of all the forces is zero. This statement is called the first condition for equilibrium. This condition is satisfied if the vector polygon representing all the external forces acting on the body is a closed figure. Analytically this condition is satisfied if each set of rectangular components of the forces separately adds to zero, or Rx = Fx = 0 (5) Ry = Fy = 0 (6)

Mechanical equilibrium A standard definition of static equilibrium is: A system of particles is in static equilibrium when all the particles of the system are at rest and the total force on each particle is permanently zero. [1]

This is a strict definition, and often the term "static equilibrium" is used in a more relaxed manner interchangeably with "mechanical equilibrium", as defined next. [2] A standard definition of mechanical equilibrium for a particle is: The necessary and sufficient conditions for a particle to be in mechanical equilibrium are that the net force acting upon the particle is zero. [3]

The necessary conditions for mechanical equilibrium for a system of particles are: (i)The vector sum of all external forces is zero; (ii) The sum of the moments of all external forces about any line is zero. [3]

As applied to a rigid body, the necessary and sufficient conditions become: A rigid body is in mechanical equilibrium when the sum of all forces on all particles of the system is zero, and also the sum of all torques on all particles of the system is zero. 40 A rigid body in mechanical equilibrium is undergoing neither linear nor rotational acceleration; however it could be translating or rotating at a constant velocity. However, this definition is of little use in continuum mechanics, for which the idea of a particle is foreign. In addition, this definition gives no information as to one of the most important and interesting aspects of equilibrium states their stability.

An alternative definition of equilibrium that applies to conservative systems and often proves more useful is: A system is in mechanical equilibrium if its position in configuration space is a point at which the gradient with respect to the generalized coordinates of the potential energy is zero. 41 Lecture 12 : Free Body Diagram Free body diagram Block on a ramp (top) and corresponding free body diagram of just the block (bottom).A free body diagram is a pictorial representation often used by physicists and engineers to analyze the forces acting on a free body. A free body diagram shows all contact and non-contact forces acting on the body. Drawing such a diagram can aid in solving for the unknown forces or the equations of motion of the body. Creating a free body diagram can make it easier to understand the forces, and moments, in relation to one another and suggest the proper concepts to apply in order to find the solution to a problem. The diagrams are also used as a conceptual device to help identify the internal forces for example, shear forces and bending moments in beams which are developed within structures. Construction A free body diagram consists primarily of a sketch of the body in question and arrows representing the forces applied to it. The selection of the body to sketch may be the first important decision in the problem solving process. For example, to find the forces on the pivot joint of a simple pair of pliers, it is helpful to draw a free body diagram of just one of the two pieces, not the entire system, replacing the second half with the forces it would apply to the first half. What is included The sketch of the free body need include only as much detail as necessary. Often a simple outline is sufficient. Depending on the analysis to be performed and the model being employed, just a single point may be the most appropriate.

All external contacts, constraints, and body forces are indicated by vector arrows labeled with appropriate descriptions. The arrows show the direction and magnitude of the various forces. To the extent possible or practical, the arrows should indicate the point of application of the force they represent. Only the forces acting on the object are included. These may include forces such as friction, gravity, normal force, drag, or simply contact force due to pushing. When in a non-inertial reference frame, fictitious forces, such as centrifugal force may be appropriate. A coordinate system is usually included, according to convenience. This may make defining the vectors simpler when writing the equations of motion. The x direction might be chosen to point down the ramp in an inclined plane problem, for example. In that case the friction force only has an x component, and the normal force only has a y component. The force of gravity will still have components in both the x and y direction: mgsin in the x and mgcos in the y, where theta is the angle between the ramp and the horizontal. 42 What is excluded All external contacts and constraints are left out and replaced with force arrows as described above. Forces which the free body applies to other objects are not included. For example, if a ball rests on a table, the ball applies a force to the table, and the table applies an equal and opposite force to the ball. The FBD of the ball only includes the force that the table causes on the ball. Internal forces, forces between various parts that make up the system that is being treated as a single body, are omitted. For example, if an entire truss is being analyzed to find the reaction forces at the supports, the forces between the individual truss members are not included. Any velocity or acceleration is left out. These may be indicated instead on a companion diagram, called "Kinetic diagrams", "Inertial response diagrams", or the equivalent. Assumptions The free body diagram reflects the assumption and simplifications made in order to analyze the system. If the body in question is a satellite in orbit for example, and all that is required is to find

its velocity, then a single point may be the best representation. On the other hand, the brake dive of a motorcycle cannot be found from a single point, and a sketch with finite dimensions is required. Force vectors must be carefully located and labeled to avoid assumptions that presuppose a result. For example, in the accompanying diagram of a block on a ramp, the exact location of the resulting normal force of the ramp on the block can only be found after analyzing the motion or by assuming equilibrium. Other simplifying assumptions that may be considered include two-force members and threeforce members. 43 Lecture 13 : Equation Of Equilibrium Equation of equilibrium When a body is in equilibrium, resultant of all forces and moments acting on that body is zero. Stated in another way, a body is in equilibrium if all forces and moments applied to it are in balance. These requirements are contained in the vector equations of equilibrium which in two dimensions may be written in scalar from as FX = 0, FY = 0, MO =0 The third one represents the zero sums of the moments of all forces about any point O on or off the body. Let's look at a truss PQS AB CD

FBD 44 PQS A x

CD W A y B y yy xx Ay FA FA MB = => = => = => 0 0 0 Additional equations could be written. = 0 MB Does not provide any new info. This is not an independent equation. We can use = 0 MB to replace one of the above 3. Example 1.Given Find: Reactions 2m 2 kN

2 kN 1.5 m 1.5 m AB 4 kN 220 0 = ++= = Ax Ax x R R F 30 0 ++= = Ay B y RR F45 FBD Substituting RB into y-equation: RAy = -1.5 KN

Example2.Given

Find: Reactions at A FBD 2m 2 kN 2 kN 1.5 m 1.5 m AB R B R Ay R Ax 3 ft 2 ft 2000 lbs ACB 3000 ft lbs 3 ft 2 ft 2000 lbs ABC 3000 ft lbs R

Ay R Ax MA 0 0 = = Ax x R F 2000 lbs 2000 0 0 = = = Ay Ay y R R F ,7 000 ft lbs 3000 2000 )5( 0 0

= + = = A A A M M M46 A simple free body diagram, shown above, of a block on a ramp illustrates this. All external supports and structures have been replaced by the forces they generate. These include: Mg: the product of the mass of the block and the constant of gravitation acceleration: its weight. N: the normal force of the ramp. Ff: the friction force of the ramp. The force vectors show direction and point of application and are labeled with their magnitude. It contains a coordinate system that can be used when describing the vectors. Example 3. Two loads 400N and 500N are suspended in a vertical plane by three springs as shown in Figure. Find the tension in the strings 47 Fig for Example.1 Obviously tension in OB = TOB = 500N Resolving the forces at O vertically POA.sin30 0 + POB.sin30

o = 400N Or, POA = 300N Resolving the forces at O horizontally POC + POA.cos30 o = POB.cos30 O or, POC + 300N(cos30 o ) = (500N)cos30 o POC = 100 3 N Tension in OA = 300N Tension in OB = 500N Tension in OC = 100 3 N Assignment Problem. 1 An electric light fixture weighing 15 Newton hangs from a point C, by two strings AC and BC. AC is inclined at 60 to the horizontal and BC at 45 to the vertical as shown in Fig. /Prob.1.Using Lami s theorem or otherwise determine the forces in the strings AC and BC. 400N 500N B C O 30 30 A48

15N Example2 Given: The loading car weight is 5500 lbs and its CG is at point G. Find: tension in cable and reactions at wheels. Multiple Choice Questions 1. Free body diagram can be applied only in a) Dynamic equilibrium problem b) Static equilibrium problem c) Both static & dynamic equilibrium problems d) None of these 2. If the body is in equilibrium ,we may conclude that a) No force acting in it b) Moment of all forces about any point is zero c) The resultant of all forces acting on it is zero d) Both b&c B A 45 O 60 O C 30" 25 o G 25" 24"

25" T49 3. The algebraic sum of the moments of two forces about any point in their plane is equal to the moment of their resultant about that point is known as a) Principle of moments b) Varignon s theorem c) Lamis theorem d) None of these 4. Two coplanar couples having equal and opposite moments a) Produce a couple and an unbalanced force b) Are equivalent c) Balance each other d) None of these 5. A free body diagram of a body represents a) With its surroundings and external forces acting on it b) Isolated from its surroundings and all external forces acting on it c) Isolated from all external actions d) None of these 6.The force which meet at one point and their lines of action also lie on the same plane are known as forces.

a) Coplanar concurrent b) Coplanar non-concurrent c) Non-coplanar non-concurrent d) None of these 50 Lecture 14 : Friction Friction is the force distribution at the surface of contact between two bodies that prevents or

impedes sliding motion of one body relative to the other. This force distribution is tangent to the contact surface and has, for the body under consideration, a direction at every point in the contact surface that is in opposition to the possible or existing slipping motion of the body at that point. Types of friction Dry friction resists relative lateral motion of two solid surfaces in contact. Dry friction is also subdivided into static friction between non-moving surfaces, and kinetic friction (sometimes called sliding friction or dynamic friction) between moving surfaces. Lubricated friction or fluid friction resists relative lateral motion of two solid surfaces separated by a layer of gas or liquid. Fluid friction is also used to describe the friction between layers within a fluid that are moving relative to each other. Skin friction is a component of drag, the force resisting the motion of a solid body through a fluid. Internal friction is the force resisting motion between the elements making up a solid material while it undergoes deformation. 51 Consider a solid block of mass m resting on a horizontal surface as shown. Assume that the contacting surfaces are rough. As we gradually increase the load, the block remains static till the load reaches a threshold value. Since the force in the x-direction has to be balanced, it is apparent that as the magnitude of F increases from zero, the friction force also increases. Friction force, is thus, self adjusting. However, the friction force cannot increase beyond a limit. Thus there is a limiting value of friction. The maximum value of friction force, which comes into play, when the motion is impending, is known as limiting friction. When the applied force is less than the limiting friction, the body remains at rest and such frictional force is called static friction, which may have any value between zero and the limiting friction. If the value of the applied force exceeds the limiting friction, the body starts moving over the other body and the frictional resistance experienced by the body while moving is known as Dynamic friction. Dynamic

friction is found to be less than limiting friction. See the following animation to understand the phenomenon of dry friction. It is experimentally found that the magnitude of limiting friction bears a constant ratio to the normal relation between the two surfaces and this ratio is called coefficient of Friction. Coefficient of friction = = Where F is limiting friction and N is the normal reaction between the contact surfaces. 52 Columb Laws of friction: (1) The force of friction always acts in a direction opposite to that in which the body tends to move. (2) Till the limiting value is reached, the magnitude of friction is exactly equal to the force which tends to move the body. (3) The magnitude of the limiting friction bears a constant ratio to the normal reaction between the two surfaces. (4) The force of friction depends upon the roughness/smoothness of the surfaces. (5) The force of friction is independent of the area of contact between the two surfaces. Angle of static friction: Consider the block on the following surface. The free body diagram is shown. The direction of resultant R measured from the direction of N is specified by tan a=F/N. When the friction force reaches its limiting static value Fmax, the angle a reaches a maximum. 53 Value of fs. Thus tan fs = ms The angle fs is called the angle of static friction. Angle of kinetic friction: When slippage is occurring, the angle a has a value fR corresponding to the kinetic friction force. tan fR = mR Cone of friction:

When a body is having impending motion in the direction of P the frictional force will be the limiting friction and the resultant reaction R will make limiting friction angle a with the normal as shown in the following figure. If the body is having impending motion in some other direction, again the resultant reaction makes limiting frictional angle a with the normal in that direction. Angle of Repose: The maximum inclination of the plane on which a body, free from external forces, experiences repose (sleep) is called Angle of Repose. 54 Now consider the equilibrium of the block shown above. Since the surface of contact is not smooth, not only normal reaction, but frictional force also develops. Since the body tends to slide downward, the frictional resistance will be up the plane. forces normal to the plane =0, gives N=Wcos forces normal to the plane =0, gives F=Wsin 2 1

Dividing equ (2) by equ (1), we get

If N is the value of normal force when motion is impending, frictional force will be

N and hence Hence, to avoid free sliding, the inclination angle should be less than the friction angle.

55 Lecture 15 : Coefficient Of Friction Coefficient of friction The coefficient of friction (COF), also known as a frictional coefficient or friction coefficient, symbolized by the Greek letter

, is a dimensionless scalar value which describes the ratio of the force of friction between two bodies and the force pressing them together. The coefficient of friction depends on the materials used; for example, ice on steel has a low coefficient of friction, while rubber on pavement has a high coefficient of friction. The coefficient of friction is an empirical measurement it has to be measured experimentally, and cannot be found through calculations. Rougher surfaces tend to have higher effective values. Most dry materials in combination have friction coefficient values between 0.3 and 0.6. Values outside this range are rarer, but teflon, for example, can have a coefficient as low as 0.04. A value of zero would mean no friction at all, an elusive property even magnetic levitation vehicles have drag. Rubber in contact with other surfaces can yield friction coefficients from 1 to 2. Occasionally it is maintained that is always < 1, but this is not true. While in most relevant applications < 1, a value above 1 merely implies that the force required to slide an object along the surface is greater than the normal force of the surface on the object. For example, silicone rubber or acrylic rubber-coated surfaces have a coefficient of friction that can be substantially larger than 1. Both static and kinetic coefficients of friction depend on the pair of surfaces in contact; their values are usually approximately determined experimentally. For a given pair of surfaces, the coefficient of static friction is usually larger than that of kinetic friction; in some sets the two coefficients are equal, such as teflon-on-teflon. Kinetic friction Kinetic (or dynamic) friction occurs when two objects are moving relative to each other and rub together (like a sled on the ground). The coefficient of kinetic friction is typically denoted as k, and is usually less than the coefficient of static friction for the same materials. Examples of kinetic friction: Kinetic friction is when two objects are rubbing against each other. Putting a book flat on a desk and moving it around is an example of kinetic friction. 56

Fluid friction is the interaction between a solid object and a fluid (liquid or gas), as the object moves through the fluid. The skin friction of air on an airplane or of water on a swimmer are two examples of fluid friction. This kind of friction is not only due to rubbing, which generates a force tangent to the surface of the object (such as sliding friction). It is also due to forces that are orthogonal to the surface of the object. These orthogonal forces significantly (and mainly, if relative velocity is high enough) contribute to fluid friction. Fluid friction is the classic name of this force. This name is no longer used in modern fluid dynamics. Since rubbing is not its only cause, in modern fluid dynamics the same force is typically referred to as drag or fluid resistance, while the force component due to rubbing is called skin friction. Notice that a fluid can in some cases exert, together with drag, a force orthogonal to the direction of the relative motion of the object (lift). The net force exerted by a fluid (drag + lift) is called fluid dynamic force (aerodynamic if the fluid is a gas, or hydrodynamic if the fluid is a liquid). Application of Friction 1) Wedges Wedges are small pieces of material with two of its opposite surfaces not parallel. They are used to lift heavy blocks, machinery, precast beam etc., slightly, required for final alignment or to make place for inserting lifting devices. The weight of the wedge is very small compared to the weight lifted. Hence, in all the problems, weight of wedges may be neglected. The following figure is showing a wedge: Free body diagram of the weight to be lifted is shown below: 57 The body is acted upon by three forces: weight W, R1 (the resultant of normal wall reaction and friction force) and R2 ( the resultant of the normal reaction of the wedge and the friction force). In the free body diagram, the resolved components of R1and R2 are shown by thin lines. If the friction angle is f, the R1and R2 will make angle with the respective normals to the surfaces. Note that coefficient of friction is given by . As the body is acted by three non-parallel forces, the forces must be coplanar and concurrent.

The relation between the forces can be found by Lami's theorem. The following figure shows the three forces meeting at a common point: 58 By Lami's theorem:

Now, let us make the free body diagram of the wedge. Three forces acting on the wedge are shown. R1and R2 are the resultants of normal and surface forces. Hence three forces meet at a point as shown below. 59 The Lami's theorem gives us: This analysis pertains to load being lifted by the wedge. If the load is lowered, the direction of friction forces and P will reverse. The analysis is similar, except that f will be replaced by - . For , P will be positive. That is some force will be needed to lower the load. In other words, without applied P, the load W will not get lowered and the wedge is called self-locking. 60 Lecture 16 : Solved Examples Example 1 A load of 150kN rests on a rough inclined plane (angle of inclination ). It can be

just moved up the plane if a 200kN force is applied horizontally or a force 125kN applied parallel to the plane. Determine the inclination of the plane, Solution: See Fig.1 & 2 and the coefficient of friction,.

Fig.1 to Example

200kN

150kN 125kN

150kN61

Fig.2 to Example FBD when 200kN force is applied

150kN P = W tan ( + ) Or, 200kN = (150kN) tan ( + ) Or, tan ( + ) = 200/150 = 1.333 Or, ( + ) = 54.31 0 When 125kN force is applied Sin ( + )

R1

200kN

F1

F2 R2 125kN62 P=W Cos

Or, 125 (KN) = 150kN Or, o . Therefore, o - 16.3 o = 36.8 o (Ans). = tan o = 0.292 (Ans.) Example2. Two blocks with masses mA = 20 kg and mB = 80 kg are connected with a flexible cable that passes over a frictionless pulley as shown in Fig. P9-4. The coefficient of friction between the blocks is 0.25. If motion of the blocks is impending, determine the coefficient of friction between = tan 16.3 = 53.1 = 16.3

block B and the inclined surface and the tension in the cable between the two blocks. Sin (53.1 o ) Cos Coefficient of Friction 150kN63

FY = An - 196.14 cos (35) = 0 An = 160.67 N For impending motion of the block A: (Af = ms . An) + Fx = -T + 0.25(160.67) + 196.14 sin 35 = 0

T = 152.67 N 64 Block B weight: WB = mB . g = 80 (9.807) = 784.56 N = Bf/Bn= 257.17/803.34 = 0.320

Free body diagram for block B: Fy = Bn - 160.67 - 784.56 cos 35 = 0 Bn = 803.34 N Fx = - Bf - 40.17 - 152.67 + 784.56 sin 35 = 0 Bf = 257.17 N Assignment

1.Blocks A and B, of weight 50 N and 100 N, respectively rest on an inclined plane as shown in the figure. The coefficient of friction between the two blocks is 0.3 and between block A and inclined plane is 0.4. Find the value of for which either one or both the blocks start slipping. At that instant, what is the friction force between B and A? Between A and inclined plane? 65 2. The block in figure is to be moved by applying a force T to a cable which slides over a fixed pulley. Find the value of T which will cause sliding motion of the block. 3. In the figure shown, if all the contact surfaces are smooth, then the relation between P and W is a. b. c. d. 66

Multiple Choice Questions 1. Static friction is always a) Greater than dynamic friction b) Less than dynamic friction c) Equal to dynamic friction d) None of the above 2. Coefficient of friction depends on a) Area of contact b) Shape of the surfaces c) Strength of the surface d) Nature of the surface 3. Frictional force required to move the body up the plane will rest, if it makes with the inclined plane an angle a) Equal to the angle of friction b) Less than the angle of friction c) Greater than the angle of friction d) None of the above

4.Frictional resistance depends upon a) Speed of the body b) Geometrical shape of the body 67 c) Nature of contacting surfaces d) Weight of the body and nature of contacting surfaces 5. Angle of the friction is the a) Angle between normal reaction and the resultant of normal reaction and the limiting friction. b) Ratio of limiting friction and normal reaction c) Ratio of minimum friction force to friction force acting when the body is just about to move. d) Ration of minimum friction force to friction force acting when the body is in motion 6. Ratio of liming friction and normal reaction is known as a) coefficient of friction b) Angle of friction c) Sliding friction d) None of the above

7. Kinetic friction is a) The maximum force of friction when the body is about to move. b) The force of friction when the body is in motion c) The force of friction between two lubricated bodies d) None of these. 68 8. Coulomb friction is the friction between a) Bodies having relative motion b) Two dry surfaces

c) Two lubricated surfaces d) Solids and liquids 69 Lecture 17 : Distribution of Forces Distribution of forces We know that all forces are concentrated along their lines of action and at their point of application. Actually concentrated forces do not exist in the exact since every external force applied mechanically to a body is distributed over a finite contact area however small. Example Force exerted by the road on automobile tyre is applied to the tyre over its entire area of contact. It may be applicable if the tyre is soft. When analyzing the forces acting on the car as a whole, if the dimension b of the contact area is negligible compared With other pertinent dimension e.g. distance between wheels, we may replace the actual distributed contact forces by their resultant R treated as a concentrated force. There are three categories of such problems. 1. Line distribution When a force is distributed along a line As in the continuous vertical load supp-otred by a suspended cable where W

Is the force per unit length (N/m). 2. Area distribution When a force is distributed over an area b

W70 like hydraulic pressure of water against the inner face of a section of dam. Intensity is force per unit area (N/m 2 ) which is called pressure for the action of fluid forces and stress for the internal distribution of forces in solid. 3. Volume distribution A force which is distributed over the volume of a body is called body force. The most common body force is the force of gravitational attraction, which acts on all elements of mass in a body. The determination of the forces on the supports of the heavy cantilevered structure for example accounts for the distribution of gravitational force throughout the structure. The intensity of gravitational force is the specific weight g. Where is the density (mass per unit volume) and g is acceleration due to gravity. The unit is (kg/m 3 ) (m/sec 2 ) =N/m 3

. Volume distribution of forces71 Centroid Centroid is the geometrical centre of a plane area or it is a point in a plane area such that the moment of inertia of that area about any axis through that point is zero. Centroid of a Line

The coordinates for the centroid of a line can be determined by using three scalar equations,

Centroid of an Area Centroid of an Area The centroid of an area can be determined by using three similar equations:

Centroid of a Volume72 Centroid of a Volume Similarly, centroid of a volume can be determined by

Center of Mass Three Planes of Symmetry The centroid of a volume defines the point at which the total moment of volume is zero. Similarly, the center of mass of a body is the

point at which the total moment of the body's mass about that point is zero. The location of a body's center of mass can be determined by using the following equations, 73 Center of Gravity The center of gravity of a body is the point at which the total moment of the force of gravity is zero. The coordinates for the center of gravity of an object can be determined with Here g is the acceleration of gravity (9.81m/s 2 or 32.2 ft/s 2 ). If g is constant throughout the body, then the center of gravity is exactly the same as the center of mass. 74 Lecture 18 : Determination Of Centroid

Centroid of an area: We first consider an area in a plane; let us call it the X-Y plane (see figure 1). The first moment MX of the area about the x-axis is defined as follows. Take small area element of area A and multiply it by its y-coordinate, i.e. its perpendicular distance from the X-axis, and then sum over the entire area; the sum obviously goes over to an integral in the continuous limit. Thus Similarly the first moment MY of the area about the y-axis is defined by multiplying the elemental area axis, and summing or A by its x-coordinate, i.e. its perpendicular distance from the Y-

integrating it over the entire area. Thus 75 This is shown in figure 2.

Location of centroid Let us a A is composed of a number of small areas a1,a2,a3, So A= a1+a2+a3+...an Let (x1,y1),(x2,y2) .(xn,yn) are the x1 ..an. a1

a2 a3

Coordinates of centroid of the areas a1,a2,a3, & OY. ..an with respect to OX x2 xN

an

X Moment of areas of all the strips about Y axis = a1x1+ a2x2+a3x3+ .+anxn

Let (X, Y) be the coordinate of the centroid of the whole area. So moment of area A about Y-axis=AX Now according to the principle of moments 76 AX= a1x1+ a2x2+a3x3+ n or X= aixi/A i=1 Where ax is the algebraic sum of .+anxn = aixi

the moments of areas about X-axis and A is the total area. Similarly n Y= aiyi/A i=1 If we now increase the number of

elements into which the area A is divided, simultaneously the size of each element will decrease and we can write

X= xdA/A & Y= ydA/A Centre of Gravity determination The weights of the particles comprise a system of parallel forces which can be replaced by a single equivalent resultant weight having the defined point G of application To find the coordinates of G, we must use the moment principle. For a body to be in static equilibrium, the resultant weight must be equal to the total weights of all n particles, i.e., WR = W

Centre of Gravity or Centre of Mass for a System of n Particles The sum of the moment of the weights of all the particles about the x,y and z is equal to the 77 moment of the resultant weight about these axes. Thus, to calculate the coordinate of G, we can take the sum of moments about the y axis. This will give XWR= x1w1+x2w2+x3w3+ .+xnwn .1

Similarly, by taking the summation of moments about the x axis, we will get the coordinate, i.e. YWR= y1w1+y2w2+y3w3+ .+ynwn ..2

Although the weights of the system of particles shown in Figure do not produce a moment about the z axis, we can still obtain the coordinate by imagining the coordinate system. We can imagine that the particles are fixed in the system and being rotated 90 o about the x (or y) axis, as shown in

figure. By taking the summation of the moments about the x axis, we will get

The sum of the moment of the weights of all the particles about the x,y and z is equal to the moment of the resultant weight about these axes. Thus, to calculate the coordinate of G, we can take the sum of moments about the y axis. This will give ZWR= z1w1+z2w2+z3w3+ .+znwn 3

Equations (1) to (3) can be presented in a generalized form and symbolically written in

X= xw/w, Y= yw/w, Z= zw/w

.4 .5

If we replace W=Mg & w=mg, Then X= xm/M, Y= ym/M, Z= zm/M

Center of mass is necessary to determine when we are dealing with the problems related to dynamics, i.e. the motion of matter under the influence of force. Provided that the acceleration of a body due to gravity g for every particle is constant, thus we will have W = mg. Substituting into equation 4 and by cancelling g from the denominator and numerator. 78 By comparison then, the center of gravity coincides with the center of mass. From general principles of Statics, we already know that particles will have weight only under the influence of gravitational force. The center of mass, on the other hand is independent of gravity. Axis of Symmetry Finding the centroid of a body is greatly simplified when the body has planes of symmetry. If a body has a single plane of symmetry, then the centroid is located somewhere on that plane. If a body has more than one plane of symmetry, then the centroid is located at the intersection of the planes. 79 Lecture 19 : Centroid Of An Arc

Example1. To find the centroid of an arc Locate the centroid of a circular arc as shown in figure. Choosing the axis of symmetry as the X-axis make Y=0 A differential element of arc has the length DL=rd expressed in polar coordinates and the X coordinate d of the element is rcos . now, L=2 r r rcos

We know LX =

xdL or 2 r X = -

(rcos ) rd

2 r X= 2r 2 sin or X= rsin / Y For a semicircular arc, 2 = , which gives X=2r/ Example2. To find the centroid of a triangle Determine the distance H from the base of a triangle of altitude h to the centroid of its area. y We consider an elementary strip of thickness dy 80 At a distance y from base. The width of the Strip is x. Now ABC & B b X AEF are similar. C h E F dy A

So we can write x/b =h-y/y or x=b(h-y)/y Now area of elementary strip= dA = x.dy

So dA = b(h-y)dy/y Area of the h So H = ydA/A = y b(1-y/h)dy/ (1/2) b h = 2/h[ (y 2 /2)-(y 3 /3h)] = --------0 --Thus the centroid of the area is at a distance h/3 from base or 2h/3 from the apex of the triangle. Example3: Calculate the centroid of a semicircular disc of radius R. It would be quite easy to solve this problem if the centre D of the circle is kept at the origin but we want to do the problem with the disc positioned as drawn below. The equation of OBC (the circular boundary of the disc) is 81 where R is the radius of the circle. The total area of the plate is . To calculate XC , we take a vertical strip of width dx at x and calculate With , we get To evaluate this integral, we let so that the limits of . Then which gives To calculate YC we need to calculate MX = , where dA represents as strip from x1 to x2 (see figure 6) From the equation of the circle we get This gives 82 and therefore Substituting y = R sin , we get integration are from 3 ABC =A = x b x h h

This gives Thus the centroid of the semicircle shown is at . Notice that the y coordinate of the centroid is less than which is easily understood because more of the area is concentrated towards the x-axis. We would not like to emphasize that the centroid (XC YC) gives a point fixed in a given planar surface and no matter in which co-ordinate system we calculate this point, it will always come out to be the same point in the surface. Thus it is a property of a surface.

83 Lecture 20: Solved Examples Example1: Calculate the centroid of a square of side a and on its two sides let there be two equilateral triangles stuck on it (see figure 7). We will consider this body as composed of the square AOBD, the triangle CDE on its right CDE and triangle EAD on its top. Then for the square we have For the triangle on the right of the square And for the triangle on top of the square . Thus for the entire plane we get 84 Similarly So because of the triangles, the centroid shift a bit to the right and a bit up with respect to the centroid of the square; this happens because of the added area of the triangles.

Example 2: To find the centroid of an area (ABCDE) that has been obtained by removing a semicircular area from a square. We know the position of the centroid of the square and the semicircular area. Thus

Therefore 85

From the previous calculation, we know that the centroid for semicircle is from the base. So In the present case we have The centroid of the figure ABCDE is then This is a little more than 0.25a. If we had removed a rectangular area equal to half the square, the X C for the area left would have been at 0.25a; because of the extra area to the right of this point when the semicircle is removed, the centroid shifts slightly to the right. After introducing you to the mathematical concepts of the first moment and centroid of a surface area, we now apply these ideas to problems in mechanics.

Y Assignments 1. Determine the y-coordinate of the centroid of the shaded area. Check your result for the special case a=0. h a 60 2. Determine the x and y coordinates of The trapezoidal area. a b Y 60 X 86

X h 87 Lecture 21: Moment Of Inertia Moment of Inertia (area) The Area Moment Of Inertia of a beams cross-sectional area measures the beams ability to resist bending. The larger the Moment of Inertia the less the beam will bend. The moment of

inertia is a geometrical property of a beam and depends on a reference axis. The smallest Moment of Inertia about any axis passes through the centroid. The following are the mathematical equations to calculate the Moment of Inertia The second moments of the area A about the x and y axes denoted as Ixx and Iyy respectively are defined as: y is the distance from the x axis to an infinitesimal area dA. Let x is the distance from the y axis to an infinitesimal area dA. Note that, (1) The first moment of area can be positive or negative, whereas the second moment of area is positive only. 88 (2) The element of area that is farthest from the axis contribute most to the second moment of area. There is two theorem 1) parallel axis theorem 2) perpendicular axis theorem Parallel axis theorem In the parallel axis theorem or Steiner's theorem can be used to determine the moment of inertia of a rigid body about any axis, given the moment of inertia of the object about the parallel axis through the object's center of mass and the perpendicular distance between the axes. The second moment of area or area moment of inertia about any axis is the sum of the second moment of the area about a parallel axis at centroid and is Ad 2 where d is the perpendicular distance between the axis for which I is being computed about the parallel centroid axis. A is the area. Mathematically, I about any axis = I about a parallel axis at centroid + Ad 2 A: is the cross-sectional area.

d is the perpendicular distance between the centroidal axis and the parallel axis. Let x be the axis parallel to and at a distance d from an axis x' going through the centroid of an area. The x' is the centroidal axis. The second moment of area about the x-axis is 89 As y = y'+d Simplifying the above expression The second term on the right hand side is zero, as x' is the centroidal axis. Hence, Example: Find out moment of inertia of a rectangular lamina of base b and height h Let x and y be a set of orthogonal axes passing through the centroid. X-Y axes are also the axes of symmetry. Because of this, Ixy= 0 = Iyx90 if we want to find out the moment about the bottom edge, we can use the parallel axis theorem. 91

Lecture 22: Moment Of Inertia Of Common Figures Moment of Inertia of some common fig Rectangle Moment of Inertia About x axis About y axis Moment of Inertia

Circle 92 Moment of Inertia About x axis About y axis Moment of Inertia

Triangle

Moment of Inertia about x axis = Half circle93 Moment of Inertia about x axis= Moment of Inertia about y axis= Quarter circle Moment of Inertia about x axis= Moment of Inertia about y axis= Perpendicular axis theorem The moment of inertia of a plane area about an axis normal to the plane is equal to the sum of the moments of inertia about any two mutually perpendicular axes lying in the plane and passing through the given axis Polar moment of inertia The Polar Area Moment Of Inertia of a beams cross-sectional area measures the beams ability to resist torsion. The larger the Polar Moment of Inertia the less the beam will twist. The following are the mathematical equations to calculate the Polar Moment of Inertia: 94

JZZ

x is the distance from the y axis to an infinitesimal area dA y is the distance from the x axis to an infinitesimal area dA. Using the Perpendicular axis theorem yields the following equations for the Polar Moment of Inertia: JZ = IX + IY Mass moment of inertia The Mass Moment of Inertia of a solid measures the solid's ability to resist changes in rotational speed about a specific axis. The larger the Mass Moment of Inertia the smaller the angular acceleration about that axis for a given torque. The mass moment of inertia depends on a reference axis, and is usually specified with two

subscripts. This helps to provide clarity during three-dimensional motion where rotation can occur about multiple axes. Radius of gyration It is a mathematical term & is defined by the relation as shown below. Ixx = AKx 2 = y 2 dA or Kx = (Ixx/A) Similarly, Iyy = (Iyy/A) & Izz = (Izz/A) Example 1 Calculate the moment and product of area for a quarter of an ellipse as shown in figure 6. 95 Equation of the ellipse whose quarter is shown in figure 6 is: . For calculating choose an area element parallel to the x-axis to calculate dA=x.dy and perform the integral Using the equation for ellipse, we get which gives This integral can easily be performed by substituting y = b sin and gives Similarly by taking a vertical strip to perform the integral, we calculate and get 96 Next we calculate the product of area IXY. To calculate IXY, we take a small element ( ) as shown in figure 7, multiply it by x and y and integrate to get For a given x , the value of y changes from 0 to so the integral is This integral is easily performed to get Thus for a quarter of an ellipse, the moments and products of area are ] If we put a = b, these formulas give the moments and products of area for a quarter of a circle of

radius a. I will leave it for you to work out what will be for the full ellipse about its centre. Using the second moment of an area, we define the concept of the radii of gyration. This is the point which will give the same moment of inertia as the area under consideration if the entire area was concentrated there. Thus 97 define the radii of gyration kX and kY about the x- and the y-axes, respectively. In the example of a rectangular area of size a x b with side a parallel to the x-axis, we had , . So for this rectangle, the radii of gyration are and . Having defined the moments and products of area, we now describe a relationship between the second moment of an area about a set of axes passing through the centroid of that area and another set of (x-y) axes which are parallel to those passing through the centroid. This is known a transfer theorem. 98 Lecture 23: Solved Examples . Example Determine the moment of inertia of the composite area about the x axis. The transfer formula was invented for cases such as this where a composite shape requires a single moment of inertia and the individual parts do not share their centroidal axis. Ixx=sum(Ic+Ad 2 ) In this case: Ixx = IA +AAd 2 + IB +ABd 2 Ixx = 1/12(6in)(2in) 3

+ (12in 2 )(2in) 2 + 1/12(2in)(6in) 3 + 12in 2 (2in) 2 Ixx = 4 in 4 + 48 in 4 + 36 in 4 + 48 in 4 Ixx = 134 in 4 Y X =KY 2

3 Example Determine the moment of inertia of the area under the

parabola about the x axis. Solve the problem by using a) a horizontal strip and b) a vertical strip. a)Since all parts of the horizontal strip is the same X =Ky 299 distance from the x-axis, moment of inertia of the strip about the axis is y 2 dA Where dA = (4-x) dy = 4 (1- y 2 /9) dy y

dy

4 [x =Ky 2 , so K = 4/9, so x = 4/9 y 2 ] 3 IY = y 2 dA = 4y 2 (1- y 2

/9) dy = 72/5 units 0

y = 14.40 units (ans) x dx

b) All parts of the vertical strip are at different distance from x-axis. Moment of inertia of the elemental rectangle about its base for the width dx & height y diX = 1/3 (dx) y 3 again y = 3x 1/2 /2

4 IX = 1/3 (dx) (3/2x 1/2 ) 3 = 72/5 units = 14.40 units (ans) 0 X0 Assignment R=20mm ' X

1.Find the moment of inertia about the axis of the semicircular disc. 15 Y 2. Determine the polar radius of gyration of the area of the equilateral triangle about the midpoint M of its base. X M 100 Multiple choice question X

Centre of gravity & Centroid 1. The point through which the whole weight of the body acts is known as a) Centre of percussion b) Centre of gravity c) Centre of mass d) None of the above 2. The plane figures like triangle, circle etc have only areas, but no mass. The centre of such figures is known as a) Centroid b) Centre of gravity c) Both centroid & centre of gravity d) None of the above 3. Centre of mass of a body a) Must lie somewhere inside the body b) Is located at the geometric centre of the body c) Is synonymous with centre of gravity d) Lies at the geometric centre of the body provided it is uniform density

4. Centre of gravity of the plane lamina is not at its geometrical centre if is a a) Circle b) Square c) Rectangle d) Right angle triangle

5. The distance of centre of gravity of a semicircle of radius r from the diameter is a) 3r/ 2 b) 2r/ 3 c) 3r/ 4 d) 4r/ 2 6. Centre of gravity of a quadrant of a circle lies along its central radius at a distance of a) 0.3R b) 0.44R c) 0.5 R d) 0.6 R 7. Centre of gravity of a T section 100 x 150 x50 mm from its bottom is a) 7.5 101 b) 78.5 c) 50 d) 89.5

Moment of Inertia 1. Moment of inertia of a body is the a) Moments of its inertia b) Rotational analogue of mass c) Rotational moment acting on the body

d) Inertia moment acting on the body 2. Moment of inertia about a principle axis is called a) Mass moment of inertia b) Second moment of inertia c) Principal moment of inertia d) All the above 3. Moment of inertia of a triangle of base b and height h with respect to its base would be a) bh/8 b) bh/6 c) bh/12 d) bh/3 4. Moment of inertia of a body does not depends upon a) Shape of the body b) Mass of the body and its distribution within the body c) Axis of rotation of the body d) Angular velocity of the body

5. The ratio of moment of inertia of a rectangle and that of a triangle, having same base and height with respect to their bases will be a) 2:1 b) 3:1 c) 4:1 d) 5:1 102 Lecture 24: Concept Of Stress INTRODUCTION

Analysis of stress and strain Concept of stress : Let us introduce the concept of stress as we know that the main problem of engineering mechanics of material is the investigation of the internal resistance of the body, i.e. the nature of forces set up within a body to balance the effect of the externally applied forces. The externally applied forces are termed as loads. These externally applied forces may be due to any one of the reason. (i) due to service conditions (ii) due to environment in which the component works (iii) through contact with other members (iv) due to fluid pressures (v) due to gravity or inertia forces. As we know that in mechanics of deformable solids, externally applied forces acts on a body and body suffers a deformation. From equilibrium point of view, this action should be opposed or reacted by internal forces which are set up within the particles of material due to cohesion. These internal forces give rise to a concept of stress. Therefore, let us define a stress. 103 Stress: Let us consider a rectangular bar of some cross sectional area and subjected to some load or force (in Newton s) Let us imagine that the same rectangular bar is assumed to be cut into two halves at section XX. The each portion of this rectangular bar is in equilibrium under the action of load P and the internal forces acting at the section XX has been shown Now stress is defined as the force intensity or force per unit area. Here we use a symbol to represent the stress. Where A is the area of the X section 104 Here we are using an assumption that the total force or total load carried by the rectangular bar is uniformly distributed over its cross-section. But the stress distributions may be for from uniform, with local regions of high stress known as

stress concentrations. If the force carried by a component is not uniformly distributed over its cross-sectional area, A, we must consider a small area, A which carries a small load P, of the total force P', Then definition of stress is As a particular stress generally holds true only at a point, therefore it is defined mathematically as Units : The basic units of stress in S.I units i.e. (International system) are N / m 2 (or Pa) MPa = 10 6 Pa GPa = 10 9 Pa KPa = 10 3 Pa Some times N / mm 2 units are also used, because this is an equivalent to MPa. While US customary unit is pound per square inch psi. 105 TYPES OF STRESSES : only two basic stresses exists : (1) normal stress and (2) shear shear stress. Other stresses either are similar to these basic stresses or are a combination of these e.g. bending stress is a

combination tensile, compressive and shear stresses. Torsional stress, as encountered in twisting of a shaft is a shearing stress. Let us define the normal stresses and shear stresses in the following sections. Normal stress: We have defined stress as force per unit area. If the stresses are normal to the areas concerned, then these are termed as normal stresses. The normal stresses are generally denoted by a Greek letter ( ) is also known as uniaxial state of stress, because the stresses acts only in one direction however, such a state rarely exists, therefore we have biaxial and triaxial state of stresses where either the two mutually perpendicular normal stresses acts or three mutually perpendicular normal stresses acts as shown in the figures below : 106 Tensile or compressive stresses : The normal stresses can be either tensile or compressive whether the stresses acts out of the area or into the area Bearing Stress : When one object presses against another, it is referred to a bearing stress ( They are in fact the compressive stresses ). 107 Shear stress: Let us consider now the situation, where the cross- sectional area of a block of material is subject to a distribution of forces which are parallel, rather than normal, to the area concerned. Such forces are associated with a shearing of the material, and are referred to as shear forces. The resulting force interistes are known as shear stress. The mean shear stress being equal to Where P is the total force and A the area over which it acts. As we know that the particular stress generally holds good only at a point therefore we can define shear stress at a point as 108 The Greek symbol ( tau ) ( suggesting tangential ) is used to denote shear stress. However, it must be borne in mind that the stress ( resultant stress ) at any point in a body is basically resolved into two components and one acts perpendicular and other parallel to the area concerned, as it is clearly defined in the following figure.

The single shear takes place on the single plane and the shear area is the cross - sectional of the rivet, whereas the double shear takes place in the case of Butt joints of rivets and the shear area is the twice of the X - sectional area of the rivet. CONCEPT OF STRAIN Concept of strain : if a bar is subjected to a direct load, and hence a stress the bar will change in length. If the bar has an original length L and changes by an amount L, the strain produce is defined as follows: Strain is thus, a measure of the deformation of the material and is a nondimensional Quantity i.e. it has no units. It is simply a ratio of two quantities with the same unit. 109 Since in practice, the extensions of materials under load are very very small, it is often convenient to measure the strain in the form of strain x 10 -6 i.e. micro strain, when the symbol used becomes .

Sign convention for strain: Tensile strains are positive whereas compressive strains are negative. The strain defined earlier was known as linear strain or normal strain or the longitudinal strain now let us define the shear strain. Hook's Law : A material is said to be elastic if it returns to its original, unloaded dimensions when load is removed. Hook's law therefore states that Stress ( ) strain( )

Modulus of elasticity : Within the elastic limits of materials i.e. within the limits in which Hook's law applies, it has been shown that Stress / strain = constant This constant is given by the symbol E and is termed as the modulus of elasticity or Young's

modulus of elasticity 110 Thus The value of Young's modulus E is generally assumed to be the same in tension or compression and for most engineering material has high, numerical value of the order of 200 GPa Poisson's ratio: If a bar is subjected to a longitudinal stress there will be a strain in this direction equal to E . There will also be a strain in all directions at right angles to . The final

shape being shown by the dotted lines. It has been observed that for an elastic materials, the lateral strain is proportional to the longitudinal strain. The ratio of the lateral strain to longitudinal strain is known as the poison's ratio . Poison's ratio ( ) = lateral strain / longitudinal strain

For most engineering materials the value of his between 0.25 and 0.3 111 Lecture 25: Possoin s Ration & Stress & Strain Curve Definitions-. Hardness is the property of resisting penetration. Normally, the hardness of steel varies in direct proportion (i.e. as one gets bigger so does the other and vice versa) to its strength the harder it is, the stronger it is, and vice-versa. Brittleness is the tendency of a material to fracture without changing shape. Hardness and brittleness are closely related. The harder (and therefore stronger) a metal is, the more brittle it is likely to be. Materials that are too brittle will have very poor shock load resistance. Malleability is the opposite of brittleness. The more malleable a material, the more readily it can be bent or otherwise permanently distorted. As hardness was closely related to strength, so then is malleability. Generally, the more malleable a metal, the weaker it is. Ductility much like malleability, ductility is the ability of the material to be drawn (stretched out) into thin sections without breaking. The harder and stronger a metal is, the less ductile, and vice versa.

Toughness The ability of a metal to absorb energy and deform plastically before fracturing. It is usually measured by the energy absorbed in an impact test. The area under the stress-strain curve in tensile testing is also a measure of toughness. STRESS STRAIN CURVE During testing of a material sample, the stress strain curve is a graphical representation of the relationship between stress, derived from measuring the load applied on the sample, and strain, derived from measuring the deformation of the sample, i.e. elongation, compression, or distortion. The nature of the curve varies from material to material. The following diagrams illustrate the stress strain behaviour of typical materials in terms of the engineering stress and engineering strain where the stress and strain are calculated based on the original dimensions of the sample and not the instantaneous value112 Proportionality limit Up to this amount of stress, stress is proportional to strain (Hooke's law), so the stressstrain graph is a straight line, and the gradient will be equal to the elastic modulus of the material. Elastic limit Beyond the elastic limit, permanent deformation will occur. The lowest stress at which permanent deformation can be measured. This requires a manual load-unload procedure, and the accuracy is critically dependent on equipment and operator skill. For elastomers, such as rubber, the elastic limit is much larger than the proportionality limit. Also, precise strain measurements have shown that plastic strain begins at low stresses. Yield points yield strength or yield point of a material is defined in engineering and materials science as the stress at which a material begins to deform plastically. Prior to the yield point the material will deform elastically and will return to its original shape when the applied stress is removed. Once the yield point is passed some fraction of the deformation will be permanent and non-reversible.113 Rupture Rupture or ductile rupture describes the ultimate failure of tough ductile materials loaded in

tension. Rupture describes a failure mode in which, rather than cracking, the material "pulls apart," generally leaving a rough surface. Working Stress From Stress-Strain diagram, It is easy to find out the magnitude of the stress which can be safety accepted for the design of the material below the elastic limit. The safe stress is called working stress. Information regarding mechanical properties of material is obtained by knowing the elastic limit, yield point & ultimate stress. Factor of safety The ratio of elastic limit to working stress is called factor of safety. The value of F.O.S. for steel is taken about 2 to 2.5 but for iron, concrete & wood is 4 to 6.

Example1 Three pieces of wood having 3.75cmx3.75cm square cross-section are glued together and to the foundation as shown in figure. If the horizontal force P=3000kg, what is the average shearing stress in each of the glued joint? Solution As there are two joints, the load P will be divided on both the joints. The area of each joint on which the load P acts, A= 10x3.75=37.5cm So total area= 2A= 2x37.5=75cm Now P= 3000kg So the average shearing stress = av=P/2A= 3000/75=40kg/cm (ans) P 10cm 114 Example2 A steel bar of 20mm diameter is loaded as shown in figure Find P2. Determine the stresses in

each part and total elongation of the bar. E=210 GPa. We solve the problem by the method of superimposition i.e. individual sections are in equilibrium under load as well as the entire bar. Now free body diagram of portion of bar with different cross-sections are drawn below. Load on each section is calculated as follows.. ABCD P2 10KN 300mm 30KN 40KN 115 For equilibrium, 40+10= P+30 or P= 20KN Stress in part AB= P1/A= 40000/{ /4(20)} = 127.38N/mm Stress in part BC= P2/A= 20000/{ /4(20)} = 63.69N/mm Stress in part CD= P3/A= 30000/{ /4(20)} = 95.54N/mm Total elongation= L = L1+ L2+ L3 = P1 L1/AE+P2 L2/AE +P3 L3/AE +2.1X10X (127.38X30+ 63.68X300 +95.54X300) = 0.409mm (Ans) Assignment1 In this figure a lever is attached to a spindle by means on a square key 6mmx6mm by 2.5cm long. If the averages shear stress in the key is not to exceed 700kg/cm 2 . What is the safe value of the load P applied to the end of the lever? 116 Assignment2 A prismatic steel bar having cross-sectional area A=3cm 2 is subjected to axial loading as shown in figure. Neglecting localized irregularities in stress distribution near the points of application of 300mm 300mm

the loads, find the net increase 6 kg/cm 2. 1M 1M 2M

in the length of the bar.

Assume E=2x10

Multiple Choice Question 1. A material obeys Hooke s law up to a) Plastic limit b) Elastic limit c) Yield point d) Limit of proportionality 2. After reaching the yielding stage while testing a mild steel specimen , strain a) become constant b) starts decreasing c) increases without any increase in load d) none of the above 3. Which one of the following material is highly elastic ? a) Rubber b) Brass c) Steel d) Glass 4. Poisson s ratio is the ratio of a) stress & strain b) modulus of elasticity & strain c) lateral strain & longitudinal strain d) none of the above

5. The unit of elastic modulus is the same of those of a) stress , strain & pressure b) stress, pressure& shear modulus c) force, stress & shear modulus d) none of the above 1.5T 1.5T 2T117 6. Impact strength of material is an index of its a) toughness b) tensile strength c) hardness d) brittleness

7. During a tensile test on a specimen of 1 cm cross section, max.load 8 N.and area of cross section at neck was 0.5 cm. of specimen ultimate tensile strength a) 4N / cm b) 16 N / cm c) 8 N / cm d) 20 N / cm 118 Lecture 26: Projectile A projectile is any object propelled through space by the exertion of a force which ceases after launch. A football after being kicked and a baseball after being hit could be considered projectiles. However, the word is most often used to refer to weapons designed with the appropriate size, shape and hardness, and propelled with sufficient speed, to cause damage (injury, property damage) to a person, animal or object they hit. Motive force

Arrows, darts, spears, and similar weapons are fired using pure mechanical force applied by another solid object; apart from throwing without tools, mechanisms include the catapult, slingshot, and bow. Other weapons use the compression or expansion of gases as their motive force. Blowguns and pneumatic rifles use compressed gases, while most other guns and firearms utilize expanding gases liberated by sudden chemical reactions. Light gas guns use a combination of these mechanisms. Railguns utilize electromagnetic fields to provide a constant acceleration along the entire length of the device, greatly increasing the muzzle velocity. Some projectiles provide propulsion during (part of) the flight by means of a rocket engine or jet engine. In military terminology, a rocket is unguided, while a missile is guided. Note the two meanings of "rocket": an ICBM is a missile with rocket engines. Non-kinetic effects Many projectiles, e.g. shells, contain an explosive charge. With or without explosive charge a projectile can be designed to cause special damage, e.g. fire (see also early thermal weapons), or poisoning (see also arrow poison). Kinetic projectiles Projectiles which do not contain an explosive charge are termed kinetic projectile, kinetic energy weapon, kinetic warhead or kinetic penetrator. Classic kinetic energy weapons are blunt projectiles such as rocks and round shot, pointed ones such as arrows, and somewhat pointed ones such as bullets. Among projectiles which do not contain explosives are also railguns, coilguns, mass drivers, and kinetic energy penetrators. All of these weapons work by attaining a high muzzle velocity (hypervelocity), and collide with their objective, releasing kinetic energy. Some kinetic weapons for targeting objects in spaceflight are anti-satellite weapons and antiballistic missiles. Since they need to attain a high velocity anyway, they can destroy their target with their released kinetic energy alone; explosives are not necessary. Compare the energy of TNT, 4.6 MJ/kg, to the energy of a kinetic kill vehicle with a closing speed of 10 km/s, which is 119 50 MJ/kg. This saves costly weight and there is no detonation to be precisely timed. This

method, however, requires direct contact with the target, which requires a more accurate trajectory. With regard to anti-missile weapons, the Arrow missile and MIM-104 Patriot have explosives, but the Kinetic Energy Interceptor (KEI), Lightweight Exo-Atmospheric Projectile (LEAP, see RIM161 Standard Missile 3), and THAAD being developed do not (see Missile Defense Agency). See also Hypervelocity terminal ballistics, Exoatmospheric Kill Vehicle (EKV). A kinetic projectile can also be dropped from aircraft. This is applied by replacing the explosives of a regular bomb e.g. by concrete, for a precision hit with less collateral damage. A typical bomb has a mass of 900 kg and a speed of impact of 800 km/h (220 m/s). It is also applied for training the act of dropping a bomb with explosives. [1] This method has been used in Operation Iraqi Freedom and the subsequent military operations in Iraq by mating concrete-filled training bombs with JDAM GPS guidance kits, to attack vehicles and other relatively "soft" targets located too close to civilian structures for the use of conventional high explosive bombs. A kinetic bombardment may involve a projectile dropped from Earth orbit. A hypothetical kinetic weapon that travels at a significant fraction of the speed of light, usually found in science fiction, is termed a relativistic kill vehicle (RKV). Wired projectiles Some projectiles stay connected by a cable to the launch equipment after launching it: for guidance: wire-guided missile (range up to 4000 meters) to administer an electric shock, as in the case of a Taser (range up to 10.6 meters); two projectiles are shot simultaneously, each with a cable. to make a connection with the target, either to tow it towards the launcher, as with a whaling harpoon, or to draw the launcher to the target, as a grappling hook does. Trajectory Trajectory is the path a moving object follows through space. The object might be a projectile or a satellite, for example. It thus includes the meaning of orbit - the path of a planet, an asteroid or a comet as it travels around a central mass. A trajectory can be

described mathematically either by the geometry of the path, or as the position of the object over time. In control theory a trajectory is a time-ordered set of states of a dynamical system (see e.g. Poincar map). In discrete mathematics, a trajectory is a sequence of values calculated by the iterated application of a mapping f to an element x of its source 120

Velocity of Projection: The velocity with which the particle is projected is called as velocity of projection (in m/sec). Angle of Projection: The angle between the direction of projection and horizontal direction is called as angle of projection ( ). Trajectory: The path traced by the projectile is called as its trajectory. Horizontal Range: The horizontal distance through which the projectile travels in its flight is called the horizontal range. Time of flight: The time interval during which the projectile is in motion is called time of flight. 121 Lecture 27: Projectile At Inclined Plane

We will study the case when the projectile flies over an inclined plane. A horizontal surface is a special case of an inclined plane. Let AB be a plane inclined at an angle to the horizontal as shown in the figure below. A

projectile is fired up the plane from point A with initial velocity u m/sec and an angle . Now, the range on inclined plane AB and the time of flight are to be determined. For the given projectile, Integrating both equations with respect to time, Where A1 and A2 are constants. Thus, A1=usin ( - ) 122 And A2=ucos ( - )

Hence, Integrating both the above equation with respect to time, Constants C1 and C1 are determined from the initial conditions. At t=0, x=y=0. Hence C1 and C2 both are zero Thus the equation of trajectory in parametric form is given as, Range on Inclined Plane: We have to find distance AB. With our coordinate system, we have to find x-coordinate of point B. The y-coordinate of B is zero. Hence, 123 Time of flight: When the flight is over, y=0 For that, we already found Hence, the time of flight is The angle of projection which gives the maximum range: For a fixed , R will be maximum, Where sin ( - )cos is maximum. This will be maximum, when 124 Assignment .1. A projectile is projected at an angle of 30 0 from horizontal with a velocity of 30 m/sec. At what times, the projectile will be at half the maximum attainable height? 2. A projectile flies over an inclined plane, which is inclined with horizontal at 30 0 . The angle of projection is 45 0

. Find out the range on the inclined plane. Also, find out the angle of projection in order to maximize the range. 3. A projectile is fired from point B to hit point D. What are the possible angle of projection, so that the target is hit. 125 Multiple Choice Questions Q.1. A particle is moving in a circular path with non-uniform speed. Then, a. Its velocity is necessarily tangential to the path and acceleration is normal to the path. b. Its velocity is necessarily tangential to the path, but the acceleration has normal and tangential compact. c. Its velocity has normal and tangential compact. d. Its acceleration is tangential Q.2. A particle is moving in a straight line with varying speed. At the instant when the speed is maximum, a. the acceleration must be 0 b. the acceleration must be maximum c. the acceleration must be minimum d. either the acceleration is 0 or it changes abruptly Q.3. If a particle is moving with a constant acceleration, its time versus displacement curve will be a. linear b. parabolic c. cubic d. sinusoidal Q.4. A particle moving in a straight line at a speed of 10 m/sec suddenly reverses its motion. At that instant, a. its acceleration will be infinite

b. its acceleration will be 0 c. its acceleration will be finite d. its acceleration will be negative Q.5. A particle is moving at a velocity 10 m/s in a straight line, when a constant acceleration of 10 m/s 2 is imposed on the particle. After a very long time, a. the particle will come to rest. b. the particle will keep oscillating. c. the particle will keep moving with increasing velocity in the opposite direction. d. the particle will keep moving with a constant velocity. Q.6. Compared to earth a projectile's range on the moon will be a. same b. more 126 c. less d. same or less 127 Lecture 28: Kinematics Suppose a particle is moving on a curve. Then its velocity is given by

Where represents the speed along the path and is the unit vector tangent to the path. The acceleration becomes 128 Assignment True & False

Q.1. Two particles are moving on a curve AB. One particle moves with a constant speed of 10 m/sec from A to B. The other particle moves from B to A at a constant speed of 10 m/sec. The accelerations of both the particles at certain point P between A and B will be same in direction and magnitude. Q.2. For a planar curve, the plane containing the curve is the osculating plane. Q.3. A binomial vector is the cross-product of unit tangent vector of the plane and principal normal. Q.4. A particle moving on a curve always has a component of acceleration along the tangent to the curve. Q.5. The normal component of a particle moving on a curve is 20 m/sec 2 at a point. If the speed of the particle is doubled, the normal component will become 40 m/sec 2 . Q.6. A particle is moving on a curve. In polar coordinates, its velocity will always be in direction. Q.7. A particle is moving on a plane spiral curve with a constant angular velocity . At a point where the radius of curvature is r, the magnitude of the velocity is . Problem Q.1The velocity of a particle moving along a space cur ve is M/sec2 At the instant when the particle is at the point where the radius of curvature is 2m, find out the acceleration of the particle.

Q2 At a particular instant, the magnitude of the velo city of a particle moving along a space curve is 10 m/sec. Its acceleration is 1 m/sec

2 and it makes 30 0 with the direction of the velocity. Find out the radius of curvature of the space curve at the point where particle is at the moment. 129

Q.3. A particle is rotating in a plane with a constant angular velocity of 1 rad/sec 2 . Simultaneously it moves towards center at a constant radial velocity of 10 cm/sec. At the instant when it is 2cm away from the center, find out its acceleration. 130 Lecture 29: Relative Motion Suppose two cars are moving at the same velocity. Even though they are moving with respect to an observer on the road, they are not moving with respect to each other. A passenger in one car will see the other car at the same distance. Thus, with respect to time, the other car is not having any velocity. We say that relative motion of one car with respect to other car is zero. We take up this problem in a general way.

Supposing an axes system is moving with respect to other axes system, what is the relationship between velocity in two system? X-Y is a fixed reference frame and x-y is a moving reference frame. To begin with, consider the x-y axes only translate with respect to X-Y, but do not rotate. If A is any particle. The position Vector of A as measured relative to the frame x-y is , where subscript A/B means "A relative to B" or "A with respect to B". The position of A with respect to X-Y, 131 Differentiating it, or,

Thus, the absolute velocity of a particle is the vector sum of the velocity of a particle with respect to a translating frame of reference and the velocity of the frame. Similarly, or, The absolute acceleration of a particle is the sum of the acceleration of the particle with respect to the translating frame and the acceleration of the frame.

Also, Note that have zero derivatives with respect to time as these are constant vectors. Their direction and magnitude both remain constant with respect to time. They only translate. A translating reference frame which has no acceleration is known as inertial frame. An example: A card board is moving in plane floor with a velocity with respect to some fixed X-Y axes. A particle is rotating in a circle of radius 2 units with a speed of 5 rad/sec. What is the absolute (with respect to X-Y) velocity of particle when the particle is at P?

Solution:132 Let us fix a x-y axis system at the center of the circle. With respect to x-y system velocity of P = = Thus the absolute velocity of P = With respect to x-y system, acceleration of the particle = -50 units(towards centre)

The acceleration of x-y with respect to X-Y system is zero. Hence the absolute acceleration is .

Let us consider axes system xy which rotates with respect to XY. Angular Velocity = Now 133 and parent from the following figure.

If angular velocity is denoted by the vector

Multiple Choice Question

Q.1. The velocity of a particle is in a frame of reference rotating with an angular velocity of10 rad/sec, i.e., unit vector and rotate with an angular velocity of 10 rad/sec about an axis perpendicular to and . The acceleration of the particle at t =25 is a. 5 + 24 b. -235 + 124 c. 235 + 124 d. 235 - 124 Q.2. A unit vector is rotating about an axis making an angle of 45 0 with the unit vector. The

angular speed is 10 rad/sec. The magnitude of the rate of change of the unit vector is a. 10 rad/sec b. 5 rad/sec c. 7.07 rad/sec d. 1.41 rad/sec 134 Q.3. A table is rotating at an angular speed of 1 rad/sec. An ant starts moving on the table radially outwards at a speed of 2 mm/sec. At the instant when the ant is 5 mm from the center, the magnitude of its velocity as observed by a fixed outside observer is a. 2 mm/sec b. 5 mm/sec c. mm/sec d. mm/sec Q.4. A boy is enjoying a merry-go-round, which is rotating at an angular speed of 2 rad/sec. The boy is at a distance of 1 m from the axis of the revolution.

When the body is at position A, another boy approached towards A, running at a speed of 0.01 m/sec. The magnitude of the velocity of walking boy as seen by the revolving boy is a. 0.01 m/sec b. 2 m/sec c. 2.1 m/sec d. 1.1 m/sec Q.5. A slider moves with a velocity 20 m/sec along a link rotating at 10 rad/sec. The magnitude of the coriolis component of the acceleration is a. 200 m/sec 2

b. 100 m/sec

2 c. 400 m/sec 2 d. 20 m/sec 2

135 Q.6. A hub with an attached blade rotates about a vertical axis. The front viewer (projection in vertical plane) is shown. If the blade vibrates in vertical plane, there will be a. a coriolis component of acceleration b. a coriolis component of acceleration in the vertical plane c. a coriolis component of acceleration in the vertical plane and a coriolis component of acceleration in the horizontal plane d. no coriolis component of Q.7. A hue with an attached blade rotates in a vertical plane. The top view (projection) in the horizontal plane is shown. If the blade vibrates, there will be a. no coriolis component of acceleration b. a coriolis component of acceleration along vertical axis c. a coriolis component of acceleration along radial direction d. a coriolis component of acceleration along the direction of vibration. 136

Q.8. A particle moves with constant relative velocity vr on the periphery of a disc of radius r in the clockwise direction. The disk also rotates with an angular velocity in the clockwise direction. The absolute acceleration of the particle is a.

b. c. d. none of these. Q.9. A link is rotating with an angular speed of . A particle slides on the link with a velocity of v. If the direction of rotation of link is changed, but the speed remains same, then a. the magnitude and direction of the acceleration remains same. b. the magnitude of the acceleration remains same, but the direction changes. c. the magnitude of the acceleration changes, but the direction remains same. d. both the direction and magnitude of the acceleration changes. Q.10. The velocity of particle A relative to B is a. 3 b. PROBLEMS .1. A disk of radius r is having the angular velocity and an angular acceleration of . A particle P moves in the opposite direction around the circumference with uniform relative velocity vr. Find the absolute acceleration of P. Q.2. In the figure, if the crank rotates with an angular speed , find out the acceleration of point P with respect to C, where C is the mid-point of crank AB. c. d. none of these. 137

Q.3. 138 A disk is rotating at an angular speed . A particle starts in the y direction at a speed v. Find out the absolute acceleration of the particle when it reaches the periphery. Q.4. A car is approaching at a velocity v. Find the acceleration of car relative to A. Q 2 A link is rotating with an angular velocity of 10 rad/sec and angular acceleration of 1 rad/sec

. A particle slides on the link with velocity of 10 m/sec and acceleration of 1 m/sec 2 . At the time when the particle is at a distance of 10 cm from the center of rotation of link, find out the absolute acceleration of the particle. Also, mention what are the various components of the acceleration. 139 Lecture 30: Kinetics Of Particle We know that, Thus. However O is fixed, thus ; For the common case of rotation of a rigid body about a fixed axis through its mass center G, clearly a = 0, and therefore . The resultant of the applied forces then is the couple . CENTER OF PERCUSSION: We have seen that if a body is rotating about fixed point not passing through its mass center, then the force system on the body may be represented by two forces passing through its center of mass in the normal and tangential direction, together with a moment . The moment may be eliminated if the line of action of tangential force is shifted to pass from the point Q instead of G, as shown in the figure below. This point Q is then called center of percussion. Let me make it clear (even at the cost of repetition!) in the following lines: For a general planar motion, we have these equations of motion: 140

The free body diagram and equivalent kinetic diagram are shown beside:

Now consider a non-centroidal rotation about O. If a tangential force F passes through G, then, Where at is the tangential acceleration. The angular acceleration is given by, Thus, Since the force F is passing through the G, the moment of that force is about G zero. If it is so, from where do we get the moment ? From the reaction at the pin, of course. Now, suppose a force F is applied at a point Q, which is at a distance of from O. Here k0 is the radius of gyration about O i.e. the moment of inertia about O is given by . Now 141 Hence, the tangential acceleration is Therefore, The net tangential force = F Thus, the force at O is zero. Hence, if a force is applied at the center of percussion, no reaction is developed at the fixed support. The sum of the moments of all forces about the center of percussion is zero.

Now Hence, the tangential acceleration is Therefore, The net tangential force = F Thus, the force at O is zero.

Hence, if a force is applied at the center of percussion, no reaction is developed at the fixed support. The sum of the moments of all forces about the center of percussion is zero. 142 Lecture 31: Rolling Motion

Now, we will discuss the other type of plane motion: motion of a disk or wheel rolling on a plane surface. If the disk is constrained to roll without sliding, the acceleration of its mass center and its angular acceleration or related. For a balanced disk i.e. for a disk whose mass center and geometrical center coincide, the acceleration of mass center is angular acceleration times the radius. Because the body is in plane motion, the kinetic diagram of the body consists of a horizontal force applied at the center and a couple. When a disk rolls without slipping, there is no relative motion between the point of the disk in contact with the ground and the ground itself. The friction force F will be self adjusting with the limiting value of .

When the disk rotates and slides at the same time, a relative motion exists between the point of the disk which is in contact with the ground and the ground itself and the force of friction has the magnitude , where is the coefficient of kinetic friction. In this case, however, the motion of the mass center G of the disk and the rotation of the disk about G are independent, and the acceleration of the center is not equal to the product of angular acceleration and radius. The three different cases are summarized as follows: Rolling, no sliding Rolling, sliding impending Rolling, and sliding a and independent. 143

When it is not known whether or not a disk slides, it should first be assumed that the disk rolls without sliding. If F is found smaller than, or equal to, , the assumption is proved

correct. If F is found larger than , the assumption is incorrect and the problem should be started again, assuming rolling and sliding. Now, let us solve a problem on rolling. A metal hook with a radius r is released from the rest on the incline. If the coefficient of static and kinetic friction are , determine the angular acceleration of the hook and time t for the hook to move a distance of S down the inclined. [ Figure A ] Fig A

Fig B The counterclockwise angular acceleration requires a counterclockwise moment about G, so F must be upward. Kinetic diagram: Assume that hook rolls without slipping, so that . Equations of dynamics, 144

(Taking moment about O) Elimination of F between (i) and (iii) and From the second equation The limiting force is In case, Fmax < F the assumption of pure rolling is wrong. In that case, Using 145 Thus, The time required for the center G of the hook to move a distance S from rest with constant acceleration 146

Lecture 32: Solution To Kinetic Problems

There are four approaches to the solution of kinetics problems. (A) Direct application of Newton's second law. (B) D.Alembert s principle (C) Use of work and energy (D) Solutions by impulse and momentum methods. Newton's Second Law: The acceleration of a particle is proportional to resultant force acting on it and is in the direction of this force. F = ma where F is the resultant force and a is the acceleration. This relation can be verified only experimentally. Assume the existence of a fixed primary inertial system. Newton's second law is valid in this system as well as with respect to any non-rotating reference system that translates with respect to the primary system with a constant velocity. Such a system is called an inertial system. We can also say that inertial" system is a system in which F = ma is valid. To understand the concept of inertial system, consider a cart from the roof of which a bob is suspended by a thread. 147

Whenever the cart accelerates or decelerates, the bob deflects. An observer sitting in the cart will think that bob is accelerating/decelerating without applying any force. Thus, for him the Newton's second law does not hold good. However, the fallacy is that he is applying the Newton's law in non-inertial system. One has to be careful in applying Newton's law to only an inertial frame of reference. SOLVED PROBLEMS An application of Newton's law will be illustrated by solving the dynamics of the following system:

It is desired to find out the acceleration of 30 kg mass , when the chord is being pulled by (209.81) N force. 148 Sol: We neglect friction and the mass of the pulley and make the free body diagram of 30 kg mass as well as the pulley attached with it. In that case, the tension everywhere in the chord is equal to the pull of (209.81) N. Upward force = Downward force due to gravity = Net force (in upward direction)

Using Newton's Second law, Vertical acceleration 149 Lecture 33: Solutions To Problems If instead of applying 20 9.81 N of puling force, 20 kg weight is suspended from the free end of the string, will the acceleration be same as before?

Let us solve this problem. Making the free body diagram of 20 kg mass.

Now, it will be clear from the following animation that when the 30kg mass goes up by a Distance of x , the 20 kg mass will move down by a distance of 2x. Hence, if the acceleration of 30kg mass is a (upward), the acceleration of 20kg mass will be 2a (downward). Applying Newton's law to mass of 20 kg, -T + 20 9.81 =20 2a ..........................(i) or, Making free body of 30 kg mass along with pulley and applying Newton's law

...........................(ii) 150

Solving (i) and (ii) (ii) D' Alembert's principle: Newton's second law is F = ma We can write it F + (-ma) =0

We know that F is the resultant of external forces applied on the particle. Considering (-ma) as a force, we can say that the body is in equilibrium under the action of external forces and force (-ma). This fictitious force is known as inertial force, and the artificial state of equilibrium created is known as dynamic equilibrium. The apparent transformation of a problem in dynamics to one in statics has become known as D' Alembert's principle. D' Alembert's published his work in his "Traite de Dynamique" in 1743. Inertia force is a fictitious force. Assume that a particle is rotated in horizontal plane by means of a string. 151 For an external observer, the particle is moving and it has a centripetal acceleration v 2 /r. There is a tension T which pulls the particle towards center. Newton's law can be applied and we get Now, suppose the observer is sitting in the particle, itself. For him the particle is not moving, but he is seeing that the particle is being pulled by a force T. Thus he will feel that there is an outward force that is balancing the force. The fictitious outward force is called inertial force. 152 Lecture 34: Solved Examples One example: A crate of mass M rests on a cart of mass m. The coefficient of friction between the rate and cart

is and between cart and the road is . If the cart is to be pulled by a force P, such that crate do not slip, determine: (a) the maximum allowable magnitude of P and (b) the corresponding acceleration of the cart. Sol: Making free body diagram of mass M

Vertical force balance gives, N = Mg Frictional force, F = N = Mg153 This is the maximum possible acceleration without slip. Now from the free body diagram of the crate + cart, P- (M+m)g - (M+m)a = 0 P = (M+m) g + (M+m)g =(M+m)g( + ) MULTIPLE CHOICE QUESTIONS Q.1. The following is not an inertial frame : a. A frame moving at a constant velocity of . b. A frame rotating with respect to an inertial frame with a constant angular velocity. c. A frame moving with velocity with respect to inertial frame. d. A frame at rest with respect to inertial frame. Q.2. The acceleration of a particle is . The mass of the particle is 2 kg. The magnitude of the net resultant force on the particle is a. 13 N b. 26 N c. 130 N d. 154

Q.3. The correct statement is a. The acceleration of 5 kg mass in both Fig A and Fig B is same. b. The acceleration of 5 kg mass in Fig A is lesser than in Fig. B. c. The acceleration of 5 kg mass in Fig A is more than that in Fig B. d. The net force on 5 kg mass is same in both the figures.

Q.4. A dynamical problem can be solved as a statically problem using a. Newton's third law b. D.Alembert s principle c. Impulse and momentum method d. Work-energy method. Fig A . Fig. B

Q.5. A stone is whirling in a horizontal plane at a speed v. The angle of inclination of string is a. Directly proportional to r and inversely proportional to v. b. Directly proportional to r and inversely proportional to v 2 . c. Directly proportional to v 2 and inversely proportional to r. d. directly proportional to Q.6. A stone is whirled in the vertical plane with the help of a string of length l. The stone is able

to complete a circle is 155 a. b. c. d. Q.7. The force on a body is proportional to its velocity and acts in a direction opposite to the velocity. The velocity decreases a. exponentially with time and linearly with displacement b. linearly with time and exponential with displacement c. linearly with time and linearly with displacement d. exponentially with time and exponentially with displacement Q.8. A particle is acted upon by a force of constant magnitude that is always perpendicular to the velocity of the particle. The motion of the particle is in a plane. Then, a. The particle moves in a circular path. b. The path must be non-circular c. The path need not be circular d. particle moves in a straight line only.

Q.9. An object is travelling at a constant speed. The following statement is incorrect about it. a. It may have a variable acceleration. b. It may have a constant acceleration. c. The force on the object is 0. d. The force on the object is 0 or is perpendicular to object. Q.10. A block A resting on a smooth floor and carrying block B upon it is pulled by a horizontal force. The acceleration A to cause a slip between A and B, depends on a. the masses A and B b. only on mass A

c. only on mass B d. does not depend on mass of A and B 156 PROBLEMS Q. 1 A 10 kg mass slides on a rough floor with a speed of 10 m/sec. A 2 kg mass is resting on it.

The coefficient of static friction between the 2 kg mass and 10 kg mass is 0.2. The coefficient of kinetic friction between the floor and 10 kg mass is 0.1. It is desired to stop the assembly by applying a horizontal force P, such that the entire assembly consisting of 10 kg and 2 kg mass stops in a minimum distance. However, during stopping 2 kg mass should not slip on 10 kg mass. Find out the maximum force P and minimum stopping distance.

Q.2. A mass of m kg is being pulled upward by a cable-pulley system. The cable is being pulled with a velocity of v downward. Find out the tension in the cable. 157 Q.3. Determine the maximum speed of the motorcycle, so that it does not loose contact with the surface.

Q.4. A disk is rotating at a constant angular velocity of . A mass m is kept on the disk. The coefficient of static friction between mass ans disk is . Find out the maximum angular speed , so that mass m does not slip. When the mass starts slipping, analyze its motion. Q.5. Using D.Alembert s principle, solve the inverted pendulum problems. At what acceleration, the cart should move, so that pendulum remains vertical? 158 Lecture 35: Work, Power & Energy Newton's law for a particle moving relative to an inertial reference is given by, Multiplying each side of the equation by dr as a dot product and integrating from r1 to r2 along the path of motion: Thus, the work done on the particle is equal to change in its kinetic energy. If we write Newton's law in component form, Then, 159

or, Taking the dot product of this equation with Similarly, Thus, the work done on a particle in any directions equals the change in kinetic energy associated with the component of velocity in that direction. Instead of using Newton's law, one can use energy equation. One example is provided.

If a car and a truck are moving with the same kinetic energy, and the same braking force is applied to stop them, which one will stop first, car or truck? Answer: Since both the vehicles are having same kinetic energy, change in kinetic energy will be same. Thus the work done by the braking force will be same. Given that braking force on the two vehicles is same, the distance covered by them is same. Power: Power is the time rate of doing work. Accordingly, the power P developed by a force F which does an amount of work W is 160 = F.v Suppose a man rises walking on the top of a hill and another man goes riding on a motor bike. Both have done the same amount of work, but the second man has used more power, because he would have done the same amount of work faster. Conservative force field: Force fields whose work is independent of the path are called conservative force fields. One example is Gravitational field. Suppose the particle is moving from 1 to 2 under the influence of gravity. Work done by the force is Thus, the work done is dependent on the end coordinates y1 and y2. In general, for a conservative force field F(x, y, z) along a path between positions 1 and 2, the

work is 161 Where U is a function of position of the end points and is called the potential energy function. Noticing that, Note that the potential energy function U depends on the reference xyz used or the datum used. However, the change in potential energy is independent of the datum used. Change in potential energy , of a force field is the negative of the work done by the force field on a particle in going from position 1 to position 2 along any path. For any closed path, clearly the work done by a conservative force field F is For an infinitesimal path difference dr starting from 1, F.dr = -dU Fxdx+Fydy+Fzdz Thus, or F = -grad U = where the operator is called the gradient operator and is given as follows for rectangular coordinates. grad = Thus, a conservative force field must be a function of position and expressed as the gradient of a scalar function. 162 If a force field is a function of position and the gradient of a scalar field, it must then be a conservative force field. Linear force: If the force is given by then it can be expressed as the gradient of Where a, b and c are constants. One example is spring force. If we put b=0, c=0 and a =-k The corresponding potential energy is 163 Conservation of mechanical Energy: We know that, Using the definition of potential energy,

U1 - U2 Thus, the sum of the potential energy and the kinetic energy for a particle remains constant at all times during the motion of the particle, provided the force field is conservative. 164 Lecture 36: Impulse & Momentum

Linear Momentum: The product of the mass and velocity is called linear momentum of the particle. Linear momentum G=mv Linear momentum is a vector having the same direction as the velocity. Thus, the rate of change of linear momentum is equal to the resultant force acting on the particle. Note that the direction of G and is not the same. In the component forms, we can write where dot indicates differentiation with respect to time. Impulse: The product of force and time is defined as linear impulse of the force. Suppose the resultant force (which may be a function of time t) acts from time t1 to time t2, then is the total impulse of that duration. 165 Hence the total linear impulse on a particle of mass m equals the corresponding change in the momentum mv. In the component forms If we plot, the resultant force with respect to time, as shown in the following figure, then

Thus, the total impulse between time t1 and t2 is the area below the resultant force curve from t1 to t2. Conservation of linear Momentum: Since the time rate of change of linear momentum is equal to the resultant force acting on the

particle, if there is no resultant force, the linear momentum is constant. This is the principle of conservation of momentum which is valid for the system of particles as well. Suppose, there are two particles A and B that interact during an interval of time. If the interactive forces F and -F between them are the only unbalanced forces acting on the particles during the interval, it follows that the linear impulse on particle A is the negative of linear impulse on 166 particle B. Therefore, the change in linear momentum of the particle A is the negative of the change in linear momentum of particle B. Hence, Example: Suppose a bullet of mass m strikes a block of mass M resting on a horizontal smooth floor and gets embedded into it. If the velocity of the bullet is V, find out the velocity of the (block + bullet) after the bullet has embedded into it. Solution: Applying the principle of momentum, mV=(M+m)Vf Where Vf is the final velocity. Thus, Now let us calculate the kinetic energy of the system before and after the impact. Before impact, (KE)1 = 1/2 mV 2

After impact (KE)2 = 1/2 (M+m) Vf 2

Hence the change in kinetic energy

] Thus, the Angular Impulse and Angular Momentum: The angular momentum M0 of a particle about O is defined as the moment of the linear momentum vector mv about O. Thus 167 So that, Hx = m(vzy - v yz) Hy = m(vxz - v zx) Hz = m(vyx - v xy) Here is a loss of kinetic energy. This is because the force field is not conservative If we take the moment of the forces about O, then Thus, the moment about the fixed point O of all forces acting on m equals the time rate of change of angular momentum of m about O. Also, The total angular impulse on m about the fixed point O equals the corresponding change in angular momentum of m about O.

Conservation of angular momentum: If the resultant moment about a fixed point O of all forces acting on a particle is zero during an interval of time, then its angular momentum remains constant. Also the total angular momentum for the system of the two particles remains constant during the interval. 168 An Example: A particle of mass m tied at the end of an extensible string is rotated at rad/sec along a circle of r radius over a smooth horizontal table top. The string is pulled down through a slot at the center of the table top at a

speed V. Calculate the speed of the particle when it reaches r/2 from the center. Solution: The particle acted upon by three forces, the force of gravity, the normal reaction of the table (which is equal to the force of gravity) and the tension of the string. The moment of the resultant force about the center is zero, i.e.,

Hence, according to conservation of angular momentum, the angular momentum H remains constant i.e., Now, where vr and are the radial and tangential velocities of the particle respectively. Since direction of radial velocity and position vector coincide, the above expression reduces Putting, in the above expression, we get Thus, for two different radii of the path of particle, which gives, For the given problem, r1 = r and r2 = r/2 and 169 Therefore, the angular velocity of the particle at the radius of r/2 is 4 The resultant speed of the particle is

Вам также может понравиться